Pharmacology and Medication Management - ML8

Lakukan tugas rumah & ujian kamu dengan baik sekarang menggunakan Quizwiz!

A two-year-old child is showing signs of shock. A 10 ml/kg bolus of normal saline solution is ordered. The child weighs 40 lb (18.18 kg). How many milliliters should be administered? Round your answer using a whole number.

182 The correct formula for this calculation is: 10 ml/kg x 18.18 kg = 181.8 ml Round to 182 ml

The nurse is administering penicillin V potassium to a child with cellulitis. The child weighs 27.5 lb (12.5 kg). The order reads penicillin V potassium 40 mg/kg/day PO divided every 6 hours. How many milligrams should this child receive with each dose? Record the answer as a whole number.

125 A 12.5 kg child with a prescription for 40 mg/kg/day will receive a total of 500 mg daily. Since the drug is given every 6 hours, the child will receive 4 doses each day. Therefore: Individual dose = 500 mg/day ÷ 4 doses/day = 125 mg per dose.

A client is admitted with a diagnosis of diabetic ketoacidosis. An insulin drip is initiated with 50 units of insulin in 100 ml of normal saline solution. The I.V. is being infused via an infusion pump, and the pump is currently set at 10 ml/hr. How many units of insulin each hour is this client receiving? Record your answer using whole number.

5 To determine the number of insulin units the client is receiving per hour, the nurse must first calculate the number of units in each milliliter of fluid: (50 units)/(100 mL)= 0.5 units/mL 0.5 units x 10 mL/hour=5 units/hour

The nurse is caring for a 10-year-old child who weighs 82.2 lb (37.3 kg) and is to receive 2.5 ml/kg/hr of 0.45% NaCl solution. How many milliliters per hour should this child receive? Record your answer using a whole number.

93 To calculate the rate per hour, the nurse should multiply 37.3 kg by 2.5 ml, which equals 93.25 ml/hr; this should be rounded to 93 ml/hr.

A child is given 0.5 mg/kg/day of prednisone divided into two doses. The child weighs 22 lb (10 kg). How much is given in each dose? 1.5 mg 5 mg 2.5 mg 10 mg

2.5 mg The child should receive 2.5 mg/dose. Here are the calculations: 0.5 mg/kg x 10 kg = 5 kg 5 mg/2 doses = 2.5 mg/dose

A client is to take 8 ounces of magnesium sulfate solution. The calibrations on the measuring device are in milliliters. How many milliliters should the nurse give? 8 ml 80 ml 240 ml 480 ml

240 ml To determine the amount of milliliters to give, use the following equation: 1 ounce equals 30 ml. 8 oz x 30 ml/oz = 240 ml

A nurse is caring for a child taking prednisone following a heart transplant. The child's pre-surgical weight was 25.6 lb (11.6 kg). The practitioner orders the child to receive 2 mg/kg/day divided every six hours. The oral solution comes prepared as 5 mg/5ml. How many milliliters will the child receive with each dose? Record your answer using one decimal place.

5.8 Here are the calculations: 11.6 kg x 2 mg/kg/day = 23.2 mg/day 23.2 mg/day ÷ 4 doses/day = 5.8 mg/dose 5 mg/5 ml = 1 mg/ml 5.8 mg/dose x 1 mg/ml = 5.8 ml/dose

What information should the nurse include in the discharge teaching for an adolescent client who's taking metronidazole to treat trichomoniasis? Sexual intercourse should be avoided until the medication is completed. Alcohol shouldn't be consumed while taking this medication. Milk products should be avoided since they reduce the effectiveness of the medicine. Exposure to sunlight should be limited to 1 hour per day.

Alcohol shouldn't be consumed while taking this medication. Clients should not consume alcohol for 48 hours following the last dose of metronidazole. The other choices have no effect on the client taking this medication.

What is the most important information for the nurse to include when teaching a 17-year-old female client about the adverse effects of isotretinoin? gram-negative folliculitis teratogenicity diarrhea vaginal candidiasis

teratogenicity The use of even small amounts of isotretinoin has been associated with severe birth defects. Most female clients taking this medication are prescribed hormonal contraceptives. Cleocin T, another medicine used in the treatment of acne, is associated with both diarrhea and gram-negative folliculitis. Tetracycline is associated with yeast infections.

The healthcare provider orders 2 g of ampicillin in 50 ml of D5W, to infuse I.V. piggyback (IVPB) over 30 minutes, for a client who had a right total knee replacement secondary to osteoarthritis. At what rate would the nurse set the I.V. infusion pump in milliliters per hour? Record your answer using a whole number.

100 60 ÷ 30 × 50 ml/h = 100 ml/h

The nurse is providing information to a client who is taking chlorpromazine. What is the most important information for the nurse to provide? Schedule routine medication checks. Stop taking the drug if adverse reactions develop. Reduce the dosage if feeling better. Stop taking medication when sunbathing.

Schedule routine medication checks. It is important to continually assess for adverse reactions and continued therapeutic effectiveness. The dosage should be changed if ordered by the primary care provider. While chlorpromazine can exacerbate serious sunburns, medication should not be discontinued without an order from the provider. Adverse reactions should be immediately reported to the provider.

A client with type 2 diabetes mellitus is prescribed capsaicin cream 0.075%. Which instruction should the nurse include in a teaching plan for this medication? "Apply this cream to open sores to prevent infection." "Apply this cream four times daily to decrease neuropathic pain sensations." "Apply this cream to necrotic areas of ulcers to aid in debridement." "Apply this cream daily to prevent dry skin."

"Apply this cream four times daily to decrease neuropathic pain sensations." This drug reduces amounts of substance P, which is involved in pain transmission. The nurse should teach the client to apply the cream four times daily for several weeks to help manage the pain of peripheral diabetic neuropathy. The cream does not prevent dry skin, debride, or prevent infection.

The home health nurse is visiting a client newly diagnosed with type 1 diabetes mellitus. The client reports nausea and abdominal pain. The nurse observes dehydration and dry skin. What question should the nurse ask the client? "Are you taking your insulin daily?" "What did you drink today?" "Did you weigh yourself today?" "When is the last time you had a checkup?"

"Are you taking your insulin daily?" The nurse should ask if the client is taking their insulin, as a common cause of DKA is missed insulin. Classic symptoms of diabetic ketoacidosis (DKA) include polyuria, weight loss, nausea and vomiting, altered mental status, abdominal pain, and Kussmaul respirations. The nurse should also check a blood glucose level. Asking the client what they drank, if they weighed themselves, and when they had a check-up will not help identify the cause of the current symptoms.

Hydrocodone with acetaminophen has been prescribed for a client with metastatic prostate cancer. What information is essential for the nurse to include in the teaching plan? "Constipation may develop with constant use." "You may feel more relaxed and calm." "You may develop blurred vision." "Nausea may occur."

"Constipation may develop with constant use." Constipation commonly develops with constant use of hydrocodone. The nurse should teach the client about constipation, and tell the client ways to decrease this risk, such as increasing fiber and liquids in the diet. Nausea may occur on occasion, however, it is not a severe problem, and could be related to constipation. Blurred vision and diarrhea are not associated with the use of hydrocodone with acetaminophen. Feeling relaxed and calm is a common side effect does not need medical attention. As the body adjusts to the medicine during treatment these side effects may go away.

What instruction should the nurse include when developing a discharge teaching plan for a client who has been prescribed phenytoin? "Don't stop taking the drug except with medical supervision." "You can take any phenytoin brand because all brands are the same." "Take the drug on an empty stomach." "You can consume alcoholic beverages in moderation."

"Don't stop taking the drug except with medical supervision." Abrupt cessation of phenytoin may trigger status epilepticus, so the client should be warned not to stop the drug unless approved by the provider. Taking phenytoin with food minimizes GI distress. Alcoholic beverages can decrease the drug's effectiveness. Changing phenytoin brands may alter the therapeutic effect.

The nurse is instructing a client with vulvovaginal candidiasis on the use of the prescribed nystatin vaginal tablets. Which statement indicates that the client requires additional teaching? "I will finish all the tablets even if I am feeling better." "I can get up to do other activities after inserting the medicine." "I will need to refrigerate the nystatin tablets." "I should report increased skin irritation to my doctor."

"I can get up to do other activities after inserting the medicine." The client will need to lay down for at least 30 minutes after insertion of the vaginal tablets. Refrigerating nystatin tablets, finishing all the tablets, and reporting increased skin irritation to the healthcare provider are all important interventions concerning this medication.

A client is receiving aspirin. Which statement made by the client needs follow-up? "I'll take the medication after a meal." "I can take Ginkgo biloba with aspirin." "I need to report if I have black stool." "I need to report loss of hearing in my ears."

"I can take Ginkgo biloba with aspirin." Aspirin, also known as acetylsalicylic acid, is used for mild to moderate pain, fever, inflammation, and atrial fibrillation stroke prevention. Aspirin may increase the bleeding when taken with herbal supplement Ginkgo biloba. The medication can cause gastrointestinal bleeding and ototoxicity. Nausea, vomiting, diaphoresis, and tinnitus are the earliest signs and symptoms of salicylate toxicity. Other early symptoms and signs are vertigo, hyperventilation, tachycardia, and hyperactivity. It should be taken with food especially if it causes stomach upset.

The nurse reviews information about how to take the prescribed tetracycline. Which statement by the client allows the nurse to determine that the client understands the information? "I can take tetracycline with milk and milk products." "I can take tetracycline with or without meals." "I can take tetracycline on an empty stomach with small amounts of water." "I can take tetracycline 1 hour before or 2 hours after meals with plenty of water."

"I can take tetracycline 1 hour before or 2 hours after meals with plenty of water." Tetracycline must be taken on an empty stomach to increase absorption, and with ample water to avoid esophageal irritation. Milk products impede absorption.

A client is taking fluphenazine. The nurse understands that teaching and discharge instructions are understood when the client states: "I need to plan frequent naps." "I need to stay out of the sun." "I need to double my fluids." "I can't eat cheese or eggs."

"I need to stay out of the sun." Fluphenazine is an antipsychotic drug that can cause photosensitivity and sunburn. Clients taking this drug don't need to increase fluid intake, avoid cheese or eggs, or plan rest periods

A nurse is teaching a client with glaucoma the proper technique for instilling eye drops. The nurse determines that teaching is effective when the client states: "I should instill the drop in the outer canthus." "I should instill the drop directly onto the cornea." "I should instill the drop in the lower conjunctival sac." "I should instill the drop near the opening of the lacrimal duct."

"I should instill the drop in the lower conjunctival sac." Eye drops should be placed in the lower conjunctival sac starting at the inner, not outer, canthus. Placing eye drops on the cornea causes discomfort and should be avoided. Eye drops shouldn't be placed by the opening of the lacrimal ducts to avoid systemic absorption.

A client with sickle cell disease is discussing his therapeutic regimen. Which statement by the client indicates further teaching is needed? "I should take one baby aspirin daily to help prevent sickle cell crisis." "I should avoid vacationing or traveling in areas of high altitude." "Cigarette smoking can cause a sickle cell crisis." "I should drink 4 to 6 L of fluid each day."

"I should take one baby aspirin daily to help prevent sickle cell crisis." Aspirin inhibits platelet aggregation and won't help prevent sickle cell crisis. Hydroxyurea is prescribed for some people to help prevent sickle cell crisis. High altitude increases oxygen demand and therefore can also precipitate a crisis. Tobacco, alcohol, and dehydration can precipitate a sickle cell crisis and should be avoided.

A nurse is about to give a client with type 2 diabetes mellitus the prescribed insulin before breakfast on the first day postpartum. Which client statement indicates an understanding of insulin requirements immediately postpartum? "I will need less insulin now than during my pregnancy." "I will need more insulin now than during my pregnancy." "I will probably be able to control my diabetes with diet and exercise now." "I will need more insulin now than before I was pregnant."

"I will need less insulin now than during my pregnancy." Postpartum insulin requirements are usually significantly lower than requirements during pregnancy. Occasionally, clients may require little or no insulin during the first 24 to 48 hours postpartum. Management of type 2 diabetes includes healthy eating, regular exercise, possibly diabetes medication or insulin therapy, and blood sugar monitoring. However, there is no way of knowing if the client will now be able to control the diabetes without insulin.

The nurse is caring for a client being treated for pedophilia. The client discloses that the dose of medroxyprogesterone is not helping to reduce sexual impulses. What is the nurse's most appropriate response? "I will review your lab results and medication dosage." "How are you tolerating that hormone therapy?" "That is an off-label use for that medication." "Have you registered yet as a sex offender?

"I will review your lab results and medication dosage." The nurse should reinforce that testosterone suppression can take from 3 to 10 months to realize symptom relief. It is important to understand serum levels as well as dosage before contacting the prescriber about a change in dosage. It is also helpful to learn how the client is tolerating the hormone, but this is not of primary importance. Hormone replacement therapy, as a treatment for this disorder, is not done universally. It is inappropriate to overreact about the disorder, or the provider's chosen treatment for this client.

The nurse is teaching a client with iron-deficiency anemia about ferrous gluconate therapy. Which statement, if made by the client, would indicate a correct understanding of the teaching? "I will take the medication on an empty stomach with orange juice." "I will take the medication with whole-grain cereal." "I will take the medication with a glass of milk." "I will take the medication with an antacid."

"I will take the medication on an empty stomach with orange juice." Preferably, ferrous gluconate should be taken on an empty stomach with orange juice. Ferrous gluconate shouldn't be taken with antacids, milk, or whole-grain cereals because these foods reduce iron absorption.

A nurse is providing instruction to a client undergoing treatment for anxiety and insomnia. The practitioner has prescribed lorazepam 1 mg PO t.i.d.. The nurse determines that teaching has been effective when the client states "I'll avoid sunlight." "I can drink red wine." "I'll avoid coffee." "I must eat enough salt."

"I'll avoid coffee." Lorazepam is a benzodiazepine used to treat various forms of anxiety and insomnia. Caffeine is contraindicated because it is a stimulant and increases anxiety. A client taking lorazepam should avoid alcoholic beverages. Clients taking certain antipsychotic medications should avoid sunlight. Salt intake has no effect on lorazepam.

The effectiveness of selective serotonin reuptake inhibitor (SSRIs) therapy, in a client with post traumatic stress disorder (PTSD), can be verified when the client states "I'm not losing my temper." "I've lost my craving for alcohol." "I'm sleeping better now." "I've lost my phobia for water."

"I'm sleeping better now." Selective serotonin reuptake inhibitors are used to treat sleep problems, nightmares, and intrusive thoughts in individuals with PTSD. Selective serotonin reuptake inhibitors are not used to control flashbacks, to treat a specific phobia, or to decrease the craving for alcohol.

The nurse is teaching a client about glaucoma treatments. What statement by a client demonstrates an understanding of the need for medication adherence? "It is important to take my medication as ordered to prevent anticholinergic effects." "I will experience diplopia if I don't take my medication as ordered." "If I don't take my medication as ordered, I will experience permanent vision loss." "I will experience pupillary constriction if I don't take my medication as ordered."

"If I don't take my medication as ordered, I will experience permanent vision loss." Without treatment, glaucoma may progress to irreversible blindness. Treatment won't restore visual damage but will halt disease progression. Blurred or foggy vision, not diplopia, is typical in glaucoma. Miotics, which constrict the pupil, are used to treat glaucoma and to permit outflow of the aqueous humor. Clients with glaucoma should avoid medications with anticholinergic effects, but taking glaucoma medications cannot prevent these effects.

A nurse is teaching a client about tricyclic antidepressants. The nurse determines that teaching has been effective when the client states "I need to call the prescriber if I get a sore throat." "This drug causes weight loss so I need to eat properly." "I should avoid all milk and dairy products." "Improvement in my mood will take up to 28 days."

"Improvement in my mood will take up to 28 days." The client's mood may not improve until the third or fourth week of tricyclic antidepressant therapy. The client needs to be reassured that the drug works slowly. The drug does not cause weight loss. It does not interact with milk and dairy products. Tricyclic antidepressants do not cause symptoms of infection.

A client is on several medications to control hypertension. As the nurse reviews the mechanism of action for each medication, what statement does the nurse use to describe how furosemide treats hypertension? "It inhibits the angiotensin-converting enzyme." "It dilates peripheral blood vessels." "It inhibits reabsorption of sodium and water in the loop of Henle." "It decreases sympathetic cardioacceleration."

"It inhibits reabsorption of sodium and water in the loop of Henle." Furosemide is a loop diuretic that inhibits sodium and water reabsorption in the loop of Henle, thereby causing a decrease in blood pressure. Vasodilators cause dilation of peripheral blood vessels, directly relaxing vascular smooth muscle and decreasing blood pressure. Adrenergic blockers decrease sympathetic cardioacceleration and decrease blood pressure. Angiotensin-converting enzyme inhibitors decrease blood pressure due to their action on angiotensin.

The nurse has instructed a client with an eating disorder about fluoxetine, and determines that teaching has been effective when the client states: "It may take 1 to 3 weeks for this medication to be effective for me." "I cannot wait to get home so I can go for a drive in my car." "I can eat anything and anytime I want. This medication will control my eating." "I should call my provider if I have cravings for large amounts of food."

"It may take 1 to 3 weeks for this medication to be effective for me." It is important for the client to understand that fluoxetine can take 1 to 3 weeks to be effective. Fluoxetine does not control eating. Operating heavy equipment or driving may be hazardous, and should only be done only after the effects of this medication are determined. Cravings should be monitored in a food diary or discussed in treatment sessions.

Estrogen therapy has been prescribed for a male client who wishes to undergo gender reassignment surgery. The nurse determines that the client understands the purpose of therapy when he states: "The therapy will assist with cross-dressing." "I will begin menstruating as a result of this therapy." "This therapy will prevent menstruation." "It will help me develop natural breasts."

"It will help me develop natural breasts." A male who receives long-term estrogen therapy will develop female secondary sexual characteristics such as breasts. A male on estrogen won't menstruate, because he doesn't have a uterus. Estrogen has no bearing on cross-dressing. Androgens would be taken by a female to stop menstruation.

2/10180019-year-old client with mild concussion after slipping in school parking lot three hours prior. No loss of consciousness. No appreciable neurological deficits. CT scan normal. Client was preparing for discharge. Now reports a 5/10 headache. Acetaminophen PO prescribed.When offered acetaminophen, the client's parents tell the nurse that they would like their child to have something stronger. What is the nurse's best response? "Acetaminophen is strong enough for your child's mild concussion." "Stronger medications may lead to vomiting, which increases the intracranial pressure." "We avoid giving aspirin to children and young adults because of the danger of Reye syndrome." "Opioids are avoided following a head injury because they may hide a deteriorating condition."

"Opioids are avoided following a head injury because they may hide a deteriorating condition." Opioids may mask changes in the level of consciousness (LOC) that indicate increased intracranial pressure (ICP) and should therefore not be given as a first-line drug in this situation. Stating that acetaminophen is strong enough ignores the parents' concern and is not appropriate. Aspirin is contraindicated in conditions that include bleeding, as well as for children or young adults with viral illnesses due to the danger of Reye syndrome; the Reye syndrome caution does not apply here. Stronger medications could lead to vomiting but would definitely sedate the client, thereby masking changes in the client's LOC, the priority concern here.

A client with active tuberculosis (TB) has been taking combination therapy with daily doses of isoniazid, rifampin, pyrazinamide, and streptomycin for the past month. What information should the nurse reinforce with the client? "Though you feel well now, if you miss any doses, your TB symptoms will become much worse very quickly." "You should continue to attend the clinic and be observed taking the medication as part of direct observation therapy." "Since you have been treated for a full month, we can now run tests to see if the medication can be stopped." "Remember that missing even one dose of these medications can make your TB less responsive to treatment."

"Remember that missing even one dose of these medications can make your TB less responsive to treatment." It is essential that the client comply with the combination drug therapy, or resistance will develop, making the TB more difficult to treat. At no time should the client stop taking the medications without the healthcare provider's authorization. If the client was required to attend direct observation therapy, the frequency of dosing would be twice weekly, not daily. Treatment must continue for a minimum of 6 months in most cases, though pyrazinamide may be discontinued after 2 months while the other medications are continued. The client will not feel worse when missing doses, which is one of the reasons adherence of this long-term treatment can be difficult.

A client has just received a renal transplant, and has started cyclosporine therapy. What is the most important information for the nurse to share with this client? "Dizziness is common." "Report any stomach discomfort or dyspepsia." "You may have a decreased appetite." "Report any fever, a flushed feeling, or lethargy."

"Report any fever, a flushed feeling, or lethargy." Fever, a flushed feeling, or lethargy suggest an infection. The nurse should closely monitor these symptoms in clients taking cyclosporine because it is an immunosuppressive drug. This medication should not cause decreased appetite, dizziness, or stomach discomfort.

The nurse has taught a family about the medication donepezil. The nurse determines that teaching has been successful when the family states "When the Alzheimer's disease worsens, they will need to stop taking this drug." "Donepezil acts like a diuretic, so they should take it in the morning." "We'll need to figure out a schedule to get their weekly blood work done." "This drug may slow their pulse, since they have pre-existing heart disease."

"This drug may slow their pulse, since they have pre-existing heart disease." Donepezil has the potential to cause bradycardia in clients with cardiac disease. Weekly blood work is not necessary. Donepezil can be used for mild, moderate, or severe Alzheimer's disease. It does not act like a diuretic, and can cause urinary retention.

A nurse is talking to a client with bulimia nervosa about the complications of laxative abuse. Which client statement indicates an understanding of the risks? "Using laxatives prevents the absorption of essential nutrients." "Chronic laxative abuse will lead to life-long dependence." "Taking too many laxatives can lead to drug toxicity." "Laxatives can lead to complications such as ulcerative colitis."

"Using laxatives prevents the absorption of essential nutrients." A serious complication of laxative abuse is the malabsorption of nutrients, such as proteins, fats, and calcium. Laxative abuse can lead to temporary dependence but the function of the colon will return with gradual withdrawal. Though laxative abuse is associated with irritable bowel syndrome, it is not a cause of inflammatory bowel disorders such as ulcerative colitis. "Drug toxicity" is not a meaningful or accurate term related to chronic laxative use.

A client with atrial fibrillation is prescribed warfarin. How should the nurse explain the purpose of this medication to the client? "Warfarin thins your blood, making it easier for your heart to pump blood to your body." "Warfarin is a blood thinner, which puts you at an increased risk for bleeding." "Warfarin is prescribed to people with atrial fibrillation to reduce the risk of having a stroke." "This medication is prescribed to help manage the irregular heart rhythm from atrial fibrillation."

"Warfarin is prescribed to people with atrial fibrillation to reduce the risk of having a stroke." Warfarin prevents vitamin K from synthesizing certain clotting factors and is used to reduce thrombus formation in the atria of people with atrial fibrillation. This reduces the risk of stroke. Warfarin and other anticoagulants do not alter the viscosity of the blood despite being commonly referred to as "blood thinners." Warfarin does not alter the rhythm associated with atrial fibrillation. Warfarin does increase the client's risk for bleeding, but this is a side effect, not the purpose of the medication.

The home health nurse is speaking to the wife of a client with neurocognitive disorder due to Alzheimer's disease. The client has been taking donepezil. The nurse is most concerned when the caregiver states "Somehow, this medication has been making my husband sleep longer in the morning." "My husband no longer has any interest in listening to the radio with me." "Yesterday, I managed to weigh my husband, and he has lost 8 lb this month." "In the last few days, the main thing that my husband wants to eat is bread."

"Yesterday, I managed to weigh my husband, and he has lost 8 lb this month." A side effect of donepezil is weight loss, and it would be important to discuss the weight loss with the primary care provider. The desire to eat bread, the ability to sleep longer, and the lack of interest in listening to the radio are not changes related to the use of donepezil.

Five days after running out of medication, a client taking clonazepam tells the nurse, "I know I shouldn't have just stopped the drug like that, but I'm OK." What is the nurse's most appropriate response? "You have handled your anxiety, and now you know how to cope with stress." "If you're fine now, chances are you won't experience withdrawal symptoms." "Let's monitor you for problems, in case something else happens." "You could go through withdrawal symptoms for up to two weeks."

"You could go through withdrawal symptoms for up to two weeks." Withdrawal symptoms can appear after one or two weeks because the benzodiazepine has a long half-life. Looking for another problem unrelated to withdrawal isn't the nurse's best strategy. The act of discontinuing an antianxiety medication doesn't indicate that a client has learned to cope with stress. Every client taking medication needs to be monitored for withdrawal symptoms when the medication is abruptly stopped.

A client at 33 weeks' gestation is leaking amniotic fluid. She is placed on an external fetal monitor. The monitor indicates uterine irritability, and contractions are occurring every 4 to 6 minutes. The provider orders nifedipine 20 mg po now and every 8 hours until birth or contractions cease. What is the most important information for the nurse to teach this client concerning nifedipine? "This will dry your mouth and make you feel thirsty." "This medicine will ensure that you do not deliver early." "You will usually feel a fluttering or tight sensation in your chest." "You may experience nausea and some dizziness."

"You may experience nausea and some dizziness." Common side effects of nifedipine are feelings of dizziness, nausea, and headache. The other side effects listed are very uncommon for this medication. Nifedipine is administered as an initial dose of 10 to 40 mg/po, then doses may continue to prevent ongoing contractions and early birth. Nifedipine relieves bronchospasm, but the client is receiving it to reduce uterine motility.

A nurse is administering vitamin K to a preterm neonate following delivery. The medication comes in a concentration of 2 mg/ml, and the ordered dose is 0.5 mg to be given subcutaneously. How many milliliters should the nurse administer? Record your answer using two decimal places.

0.25 Use the following formula to calculate drug dosages: Dose on hand/Quantity on hand = Dose desired/X The equation is as follows: 2 mg/ml = 0.5 mg/X; X = 0.25 ml

A 13-year-old client admitted with a fractured femur had an open reduction and internal fixation (ORIF) 12 hours ago. The client is reporting pain 8 out of 10 on the numeric pain scale. The provider has ordered ketorolac 20 mg to be administered I.V. every 6 hours. The pharmacy sends over ketorolac 50 mg/1 ml. How many milliliters will the nurse administer for the correct dosage? Record your answer using one decimal place.

0.4 The formula to use is: Amount Given = Desired/Have x Volume. The amount desired is 20 mg. Divide by 50 mg (sent over by the pharmacy is the amount you have). Then multiply that amount by 1 ml (because the 50 mg is in 1 ml of solution). X = 20 mg / 50 mg x 1 ml X = 0.4 ml.

A 7-year-old boy with a fractured right femur has been admitted to the pediatric unit following a car accident. The provider orders morphine 0.02 mg/kg I.V. for severe pain control. The client weighs 52 lb (23.6 kg) and is experiencing pain of 9 out of 10 on the pain scale. What dosage of morphine should the nurse administer to this client? Record your answer using two decimal places.

0.47 Dosage calculation 23.6 kg x 0.02 mg/kg = 0.47 mg Age: ? 1 month but < 12 years: I.V./subcutaneous continuous: 0.025 to 0.206 mg/kg/hour (sickle cell or cancer pain) or 0.01 to 0.04 mg/kg/hour (postoperative pain); maximum per 24 hours: 5 mg.

A 12-year-old child sustains a moderate burn injury. The parent reports that the child last received a tetanus injection at 5 years of age. Which immunization would the nurse anticipate for this child? 250 units of tetanus immune globulin I.M. 0.5 ml of tetanus toxoid I.M. 0.5 ml of tetanus toxoid I.V. 250 units of tetanus immune globulin I.V.

0.5 ml of tetanus toxoid I.M. Tetanus prophylaxis is given to all clients with moderate to severe burn injuries if it has been longer than 5 years since the last immunization, or if there is no history of immunization. The correct dosage is 0.5 ml I.M., one time, if the child was immunized within 10 years. If it has been more than 10 years, or the child hasn't received tetanus immunization, the dosage is 250 units of tetanus immune globulin, one time. There is no I.V. form of tetanus immune globulin available.

The nurse is preparing to administer digoxin to an infant who weighs 13.7 lb (6.2 kg). The practitioner has ordered 25 mcg/kg divided in three doses with one-half of the total dose for the initial dose, then one-quarter of the total dose for each of two subsequent doses given at eight-hour intervals. The medication comes in an oral solution of 0.05 mg/ml. How many milliliters would the nurse give for the second dose? Record your answer using two decimal places.

0.78 Here are the calculations: 25 mcg/kg x 6.2 kg = 155 mcg 0.05 mg/ml x 1000 mcg/mg = 50 mcg/ml 155 mcg ÷ 50 mcg/ml = 3.1 ml (total dose) 3.1 ml ÷ 4 = 0.78 ml (one-quarter dose) The nurse would give 1.5 ml for the first dose, and then 0.78 ml for the following two doses.

The nurse receives a prescription for amoxicillin 80 mg/kg/day to be administered in two divided doses to an infant who weighs 19 lb 8 oz (9 kg). The medication is supplied as 250 mg/ml. How many milliliters should the nurse administer for one dose? Record your answer using one decimal place.

1.4 The client is to receive 80 mg/kg/day divided into 2 doses. Therefore, the client receives 40 mg/kg per dose. The client weighs 9 kg, the the client should receive 40 mg/kg × 9 kg = 360 mg per dose. The medication comes 250 mg/ml. Set up a proportion of mg to ml and solve: 250 mg / 1 ml = 360 mg / X ml 250X = 360 X = 1.44 Round to the nearest tenth: the client should receive 1.4 ml per dose.

A child with hives is prescribed diphenhydramine 5 mg/kg over 24 hours in divided doses every 6 hours. The child weighs 17.6 lb (8 kg). How many milligrams should be given with each dose? Record your answer using a whole number.

10 Multiplying 5 mg by the child's weight (8 kg) gives the amount of milligrams for 24 hours (40 mg). Divide this by 4 (doses per day), giving 10 mg/dose. 5 mg/kg x 8 kg = 40 mg; 40 mg/4 doses = 10 mg/dose

A client's labor is not progressing. After ruling out cephalopelvic disproportion, the health care provider orders 1,000 ml 0.9% normal saline with oxytocin 10 units to run at 2 milliunits/min. What is the correct infusion rate in milliliters per hour? Record the answer as a whole number

12 (1000 mL saline)/(10 units oxytocin) × 1 unit/1000 milliunits × 2 milliunits/min × 60 min/hr = 12 mL/hr.

A client with new-onset seizures of unknown cause is started on phenytoin. The healthcare provider has ordered a loading dose of 15 mg/kg I.V. to be given at a rate of 40 mg/min. What is the loading dose in milligrams if the client weighs 176 lb (80 kg)? Record your answer using a whole number.

1200 15 mg/kg x 80 kg=1,200 mg.

The supervisor is performing a chart review. The nurse can be held legally liable for which documentation? 0900 Withheld mononitrate dose. Client's blood pressure is 80/40 mmHg -BSmith, RN 1200 Administered cephalosporin. The client has an allergy to penicillin -BSmith, RN 0800 administered 2 mg hydromorphone IVP per PRN orders of 1 to 2 mg every 4 hours -BSmith, RN 0900 Withheld digoxin dose. Client's apical pulse is 56 beats/min -BSmith, RN

1200 Administered cephalosporin. The client has an allergy to penicillin -BSmith, RN There is a cross-sensitivity between cephalosporin and penicillin, and the drug should not have been given. When a dosage range is ordered, any dose in that range is acceptable. Digoxin is a cardiac glycoside that acts to improve the efficiency of the heart and may slow the heart rate and the drug should not ordinarily be given if the apical pulse is less the 60. Mononitrate is a nitrate that can cause vasodilation and should not be given when hypotension is present.

Chlorothiazide is ordered for a 1-year-old client with neonatal bronchopulmonary dysplasia (chronic lung disease). The dosage ordered is 30 mg/kg/day. The client weighs 10 kg. How much is given per dose when administered 2 times per day? Record your answer using a whole number.

150 Here are the calculations: 30 mg/kg/day x 10 kg = 300 mg/day 300 mg/2 doses = 150 mg/dose

A client, with new onset of atrial fibrillation, is receiving warfarin to help prevent thromboembolism. The client will be discharged when the warfarin reaches therapeutic levels, and when the international normalized ratio (INR) ranges from 0.5 to 1 INR 3.5 to 4 INR 2 to 3 INR 1.25 to1.75 INR

2 to 3 INR In a client with atrial fibrillation, the warfarin is at a therapeutic level when the INR ranges from 2 to 3. A range of 3.5 to 4 is too high, and increases the risk of hemorrhage. Discharge would be considered when the INR is within the therapeutic range.

A 10-year-old child is admitted with asthma. The health care provider orders a methylprednisolone loading dose of 3 mg/kg. The child weighs 30 kg. It comes as a solution of 40 mg/ml. How many milliliters should the child receive? Record your answer using two decimal places.

2.25 The child should receive 90 mg per dose. Here are the calculations: 3 mg/kg x 30 kg = 90 mg 90 mg/dose ÷ 40 mg/ml = 2.25 ml/dose

The nurse prepares to give penicillin to a client with osteomyelitis. The healthcare provider has ordered 700 mg IM. The vial is a mix-o-vial containing drug powder and sterile water for injection. When mixed together the vial contains 1 g/3.4 ml. How much should the nurse draw up to give this client? Record your answer using one decimal place.

2.4 X = (Dose Desired)/(Dose on Hand or Dose Available) × milliliters/grams X =(700 mg)/1000 × (3.4 ml)/(1 g)=2.4 ml

The provider has ordered an I.V. of 5% dextrose in lactated Ringer's solution at 125 ml/hr. The I.V. tubing delivers 10 gtt/ml. How many gtt/min should fall into the drip chamber? 20 to 21 12 to 13 10 to 11 22 to 24

20 to 21 Multiply the number of ml to be infused by the drop factor: (125 ml/hr x 10 gtt/ml) = 1,250 gtt/hr 1,250 gtt/hr ÷ 60 min/hr = 20.83 gtt/min or 20 to 21 gtt/min.

An unconscious client with moderate hyperkalemia is prescribed 50 grams of sodium polystyrene sulfonate rectally. The medication is supplied as a 15 g/60 mL suspension. How many milliliters of sodium polystyrene sulfonate should the nurse administer? Record your answer as a whole number.

200 The nurse notes that the dose on hand is 15 grams per 60 ml. Using dose on hand versus dose desired formula: (50 g/15 g) x 60 mL = 200 mL.

A nurse is verifying a medication calculation completed by a nursing student prior to administration. The adult client is to receive ampicillin 150 mg/kg/day I.V. divided in 6 even doses with a maximum dose of 12 g/day. The client's weight is 80 kg. How many mg/dose will the client receive? Record your answer using whole number.

2000 The nurse should verify that the total dosage will not exceed the maximum dosage of 12 g/day. 150 mg/kg / day x 80 kg = 12,000 mg/day ÷ 6 doses = 2,000 mg per dose

A client with deep vein thrombosis (DVT) has an IV infusion of heparin sodium infusing at 1,500 units/hr. The concentration in the bag is 25,000 units/500 ml. How many milliliters should the nurse document as intake from this infusion following an 8-hr shift? Record your answer using a whole number.

240 The solution has 25,000 units of medication in every 500 ml of solution, and 1,500 units are infusing each hour over 8 hours. There are several paths to finding the intake, or amount of solution, infused. Here is one approach. Calculate how many units are in each milliliter by reducing the ratio given: 25,000 units/500 ml = 50 units/1 ml. Calculate how many milliliters the client receives each hour. Multiply known ratios so that units cancel out, leaving ml/hr, and then reduce:1 ml/50 units × 1,500 units/1 hr = 1,500 ml/50 hr = 30 ml/1 hr. Multiply the hourly rate by 8 hours: 30 ml/1 hr × 8 hr = 240 ml. The intake is 240 ml of solution.

A client with pneumonia is ordered azithromycin 500 mg I.V. daily via a peripheral I.V. catheter. The medication is pre-mixed from the pharmacy in a 50 ml bag of solution to be infused over 30 minutes. The I.V. tubing delivers 15 gtt/ml. At what drip rate should the nurse set the infusion pump? Record your answer using a whole number.

25 Equation: X=50 ml ×(15 gtt)/(30 min) X=25 gtt/min

A client with heart failure is receiving furosemide 40 mg IV. The provider orders 40 mEq of potassium chloride in 100 mL of dextrose 5% in water to infuse over four hours. The client's most recent serum potassium level is 3.0 mEq/L. At what rate (in mL/h) should the nurse set the IV pump? Record the answer as a whole number.

25 Rate = (volume to be infused in mL) ÷ (total time of infusion in hours) Rate = 100 mL ÷ 4 hr = 25 mL/hr

A child with type 1 diabetes is ordered to receive 25 ml/hr of 0.9% I.V. solution. The nurse is using a pediatric microdrip chamber to administer the medication. What is the correct drip rate for this medication? Record your answer using a whole number.

25 When using a pediatric microdrip chamber, the number of ml/hr equals the number of gtt/min. If 25 ml/hr is ordered, the I.V. should infuse at 25 gtt/min.

A client who weighs 165 lb (75 kg) with a pulmonary embolus is prescribed heparin 18 units/kg/hr by IV infusion. The heparin is supplied as 25,000 units in a 500-ml bag. At what hourly rate (ml/hr) should the pump be running to deliver the prescribed amount of heparin each hour? Record your answer using a whole number.

27 Desired rate = 18 units/kg/hr x 75 kg = 1,350 units/hr Concentration of the supplied heparin = 25000 units/500 ml = 50 units per ml Required infusion rate = 1350 units/hr divided by 50 units/ml = 27 ml/hr Although heparin should be delivered via a "smart pump," in which the drug concentration already stored in the pump's library, the nurse should still be able to calculate the hourly rate and know what is a reasonable rate of infusion to protect from errors.

A child requires I.V. fluids to infuse at 27 ml/hr. The tubing delivers 60 gtt/ml. How many gtt/min should the nurse count to ensure that the fluid is safely infusing? 60 gtt/min 14 gtt/min 27 gtt/min 54 gtt/min

27 gtt/min The nurse should count 27 gtt/min. 27 ml/h x 60 gtt/ml ÷ 60 min/h = 27 gtt/min.

A client with a history of hypertension has just had a total hip replacement. The provider orders hydrochlorothiazide 35 mg oral solution po/day. The label on the solution reads hydrochlorothiazide 50 mg/5 ml. How many milliliters should the nurse pour to administer the correct dose? Record your answer using one decimal place.

3.5 The correct formula to calculate a drug dosage is: Dose on hand ÷ Quantity on hand = Dose desired ÷ X.

Which client would be most at risk for secondary Parkinson's disease caused by pharmacotherapy? 75-year-old client using naproxen for rheumatoid arthritis 30-year-old client with schizophrenia who is taking chlorpromazine 60-year-old client who is taking prednisone for chronic obstructive pulmonary disease 50-year-old client taking nitroglycerin tablets for angina

30-year-old client with schizophrenia who is taking chlorpromazine Phenothiazines such as chlorpromazine deplete dopamine, which may lead to extrapyramidal effects. The other drugs don't place the client at a greater risk for developing Parkinson's disease.

A child is prescribed high-dose aspirin as part of the therapy for Kawasaki disease. The order is for 80 mg/kg/day PO in four divided doses until the child is afebrile. The child weighs 33.1 lb (15 kg). How many milligrams is given in one dose? Record your answer using a whole number.

300 Aspirin is used for the treatment of Kawasaki disease due to its anti-inflammatory properties. The benefits of aspirin outweigh the risks in this situation. Use the following equation: First, determine how many milligrams should be given in one day: 80 mg/kg x 15 kg = 1200 mg. Then, determine how many milligrams should be given in one dose: 1200 mg/4 doses = 300 mg/dose

A client is to receive 100 mg of cefazolin following an open reduction and internal fixation for repair of a fractured femur. The pharmacy has sent 100 mg of cefazolin in 50 ml of dextrose. The medication is to be administered over 30 minutes. Calculate the drip rate (gtt/min) using a set with a drop factor of 20 gtt/ml. Record your answer using a whole number.

33 Formula for I.V. calculations:Drip rate = (Volume/Rate) x set drop factorA medication volume of 50 ml is to be administered in 30 min using a 20 gtt/ml set.X = (50 ml / 30 min) x (20 gtt/min)X = 33.33, which rounds to 33 gtt/min.

The health care provider's order reads 2 g of cephalexin daily in equally divided doses of 500 mg each. At which frequency should the nurse administer this medication? 4 times per day 8 times per day 3 times per day 6 times per day

4 times per day Two grams is equivalent to 2,000 mg. To give equally divided doses of 500 mg, divide the desired dose of 500 mg into the total daily dose of 2,000 mg. This medication should be given every 6 hours, 4 times each day.

A client, weighing 132 lb (60 kg), is to receive phenobarbital 2mg/kg/day to be given in a divided into three equal doses. How many milligrams of phenobarbital will the client receive in each dose? Record your answer using a whole number.

40 2 mg/kg?(day (total dose) x 60 kg (client^' s weight)=120 mg/day.) The total dose per day is 120 mg. Dividing 120 mg by 3 equal doses provides the final answer of 40 mg per dose.

The nurse is caring for a 12 kg child diagnosed with epiglottitis. Vancomycin 50 mg/kg/day in three divided doses is prescribed. The medication is supplied as 500 mg/100 ml. How many milliliters per dose will the nurse administer? Record your answer using a whole number.

40 The child should receive 40 ml per dose. Here are the calculations: 50 mg/kg/day x 12 kg = 600 mg/day 600 mg/day ÷ 3 doses/day = 200 mg/dose 200 mg/dose ÷ 5 mg/ml = 40 ml/dose

A client is admitted to the hospital with lower gastrointestinal bleeding. The client's hemoglobin on admission to the emergency department is 7.3 g/dl. The healthcare provider prescribes two units of packed red blood cells (RBCs) to infuse over one hour each. Each unit of packed RBCs contains 250 ml. The blood administration set has a drip factor of 10 gtt/ml. What is the flow rate in drops per minute? Record your answer using a whole number.

42 Each unit of packed RBCs contains 250 ml. Each unit is to infuse over one hr. Use the following equation: (250 ml)/(60 minutes)= 4.16 ml/minute. Multiply by the drip factor: 4.16 ml/minute x 10 gtt/ml = 41.6 gtt/minute, which rounds to 42 gtt/minute.

A client is ordered to receive 1,000 ml of 0.45% normal saline with 20 mEq of potassium chloride (KCl) over 6 hours. The infusion set administers 15 gtt/ml. How many drops per minute should this client receive? Record your answer using a whole number.

42 The flow rate is determined by the rate of infusion and the number of drops per milliliter of the fluid being administered. gtt/ml x amount to be infused/number of minutes = the I.V. flow rate 15 gtt/ml x 1,000 ml/360 min = 42 gtt/min

The nurse is administering amoxicillin/clavulanate potassium to a child with cellulitis. The provider has ordered a total of 40 mg/kg to be over 24 hours, divided into 3 equal doses. The child weighs 33 lb (15 kg), and the pharmacy has sent amoxicillin/clavulanate 200 mg/5 mL. How many milliliters per dose should the nurse administer? Record your answer using a whole number.

5 The dose is calculated by first multiplying the weight times the milligrams. It's then divided by 3 even doses. The milligrams are then used to determine the milliliters based on the concentration of the medicine. 40 mg/kg x 15 kg = 600 mg 600 mg ÷ 3 doses = 200 mg/dose 200 mg/dose ÷ 200 mg/5 mL = 5 mL/dose.

A nurse is preparing a dose of amoxicillin for a 3-year-old with acute otitis media. The child weighs 33 lb (15 kg). The dosage prescribed is 50 mg/kg/day in divided doses every 8 hours. The concentration of the drug is 250 mg/5 ml. How many milliliters should the nurse administer? Record your answer using a whole number.

5 To calculate the child's weight in kilograms, the nurse should use the following formula: 1 kg/2.2 lb = X k /33 lb X = 33/2.2 kg = 15 kg. Next, the nurse should calculate the daily dosage for the child: 50 mg/kg/day x 15 kg = 750 mg/day The medication is divided into 3 daily doses: 750 mg/day ÷ 3 doses/day = 250 mg. The drug's concentration is 250 mg/5 ml, so the nurse should administer 5 ml.

An infant who weighs 17.6 lb (8 kg) is to receive ampicillin 25 mg/kg IV in divided doses q6h. How many milligrams should the nurse administer per dose? Record the answer using a whole number.

50 The daily dose can be calculated as: 25 mg/kg/day x 8 kg = 200 mg/day. Since 4 doses are given each day, an individual dose is: 200 mg/day ÷ 4 doses/day = 50 mg.

A toddler weighing 27.6 lb (12.5 kg) is to receive 4 ml/kg/hr of intravenous normal saline solution. The nurse will administer the fluid using microdrip tubing that delivers 60 gtt/ml. How many milliliters per hour should this client receive? Record your answer using a whole number.

50 To calculate the rate per hour for the infusion, the nurse should multiply 12.5 kg by 4 ml/kg/hr, which equals 50 ml/hr. The fact that the nurse is using microdrip tubing does not influence the calculation of the rate in milliliters per hour.

A nurse is preparing to administer the first dose of tobramycin to an adolescent with cystic fibrosis. The order is for 3 mg/kg I.V. daily in 3 divided doses. The client weighs 110 lb (50 kg). How many milligrams should the nurse administer per dose? Record your answer using a whole number.

50 To perform this dosage calculation, the nurse should calculate the client's daily dose using this formula: 50 kg x 3 mg/kg = 150 mg The nurse should then calculate the divided dose: 150 mg/3 doses = 50 mg/dose

After undergoing small-bowel resection, a client is prescribed metronidazole 500 mg I.V. The mixed I.V. solution contains 100 milliliters. A nurse is to run the drug over 30 minutes. The drip factor of the available I.V. tubing is 15 gtt/ml. What is the drip rate? Record your answer using a whole number.

50 Use the following equation: (100 ml x 15 gtt/ml) / 30 min = 50 gtt/min

A client with symptoms of acute asthma is ordered I.V. aminophylline 350 mg in 100 ml to be administered over 30 minutes. The nurse has vials of I.V. aminophylline labeled 250 mg/5 ml. How many milliliters of fluid contain the dose ordered? Record your answer using a whole number.

7 Equation: (350 mg)/X=(250 mg)/(5 ml) X=7 ml

The nurse will administer a dosage of captopril at 1.5 mg/kg/day, in divided doses, q12h, to an infant who weighs 10 kg. How much would the nurse give per dose? Record your answer using one decimal place.

7.5 Here is the calculation: 10 kg x 1.5 mg/kg/day = 15 mg/day. 15 mg/day ÷ 2 doses/day = 7.5mg/dose.

A 3-year-old child weighing 34.2 lb (15.5 kg) is to receive 5 ml/kg/hr of dextrose 5% in normal saline solution. At what rate (in ml/hr) should the nurse set the infusion pump? Record your answer using a whole number.

78 To calculate the rate per hour for the infusion, the nurse should multiply 15.5 kg by 5 ml, which equals 77.5 ml/hr, which should be rounded to 78 ml/hr.

A nurse is preparing to administer beractant to a preterm infant. The order is for 4 ml/kg. The neonate weighs 2,000 g. How many total milliliters will be used for 1 dose? Record your answer as a whole number.

8 First, convert the weight from grams to kilograms. 1,000 g = 1 kg; 2,000 g = 2 kg Then, determine how many milliliters are needed by using the following formula: 4 ml/kg x 2 kg = 8 ml total dose

A client with a positive skin test for tuberculosis (TB) is not showing signs of active disease and is being treated with isoniazid, 300 mg daily. The nurse explains to the client that the medication should be taken for 2 to 4 weeks. 3 to 6 months. 9 to 12 months. 10 to 14 days.

9 to 12 months. Because of the increasing incidence of resistant strains of TB, the disease must be treated from 9 to 12 months, or up to 24 months in some cases. Isoniazid is the most common medication used for the treatment of TB, but other antibiotics are often added to the regimen to obtain the best results.

A nurse is caring for a client with a prescription for hydromorphone 2 mg I.V. as needed for pain. The client is reporting pain at a 10/10 and is requesting to be medicated. When trying to retrieve the prescribed hydromorphone, the nurse is unable to access the computerized automated dispensing cabinet (ADC) because of an expired password. What is the most appropriate action by the nurse? Explain to the client there will be a delay in receiving pain medication and then contact information services to reset password. Administer the medication to the client after having another nurse obtain the medication from the computerized ADC. Ask another nurse for their user name and password and then obtain the medication from the computerized ADC. Ask another nurse to administer the medication to the client while contacting information services to reset password.

Ask another nurse to administer the medication to the client while contacting information services to reset password. Managing pain in a client with a pain rating 10/10 is a priority and care should not be delayed. The nurse should ask another nurse to medicate the client while contacting information services to reset the password as soon as possible to prevent delay in care for other clients. Hydromorphone is a Schedule II controlled substance and by federal law a record must be kept of the name of the nurse who obtained and administered the substance. The nurse who obtains the medication from the computerized ADC is the nurse who is required to administer it. A nurse's user name and password is a secure identification code equivalent to a nurse's signature and should never be given to another staff member.

A depressed client, who is taking fluoxetine each afternoon, tells the nurse that they have difficulty sleeping at night, is often sleepy during the day, and does not feel like doing anything. What is the nurse's best response? Tell the client to stop taking the drug until they see their health care provider Ask the prescriber whether the medication can be given early in the day Advise the client to see another provider to obtain another opinion Advise the client to continue taking the drug to see whether these effects wear off

Ask the prescriber whether the medication can be given early in the day A common side effect of fluoxetine is insomnia, which is best addressed by administering this medication early in the day. It is inappropriate for the nurse to tell the client to stop taking the drug, to continue taking it until the undesired effects wear off, or to seek a second opinion.

A toddler taking penicillin for acute otitis media developed a maculopapular rash 24 hours ago after 3 days of therapy. The parents report no other abnormal symptoms. The nurse takes what initial action? Administer albuterol (salbutamol) nebulizer. Reassure the parents that this is a mild reaction. Assess chest sounds and oxygen saturation. Administer epinephrine intramuscularly.

Assess chest sounds and oxygen saturation. It is relatively common for children to experience delayed hypersensitivity reactions to penicillin that are isolated to cutaneous eruptions. Often, it is safe for these children to receive penicillins in the future. However, the nurse must ensure this current reaction is not more serious than it appears. Because a toddler cannot adequately communicate symptoms, the nurse assesses the client's respiratory status to ensure there is no evidence of bronchoconstriction that could suggest anaphylaxis. Once a full assessment has been completed, the nurse can then request the appropriate treatments be initiated.

The nurse is caring for a client with a prescribed fentanyl 25 mcg/hr transdermal patch placed 72 hours prior. After performing safe medication checks, list in order the steps the nurse will take in placing a new patch. All options must be used.

Assess the client's vital signs and pain level. Identify and prepare a new application site. Remove the used patch and fold its adhesive edges together. Remove the new patch from its packaging and remove the backing from the adhesive layer. Press the new patch firmly to the application site with the palm of the hand for 30 seconds. Fentanyl is a Schedule II controlled substance that can cause decreased level of consciousness and respiratory depression. The nurse should closely monitor vital signs and pain levels and should assess these prior to administering a new patch. After assessment, the nurse selects a flat site on the upper body that is different from the site of the patch currently in use. The nurse then removes the used patch, folds the adhesive edges together, and either flushes the used patch down the toilet or places it in a pharmaceutical waste container as per facility policy. (Proper disposal of the used patch is necessary to prevent the misuse of the remaining medication in the used patch.) The nurse then removes the new patch from the package, removes the backing from the adhesive layer, and immediately places the patch on the prepared site. The nurse then presses the patch in place for at least 30 seconds, making certain the patch adheres with the edges sealed.

Which assessment should a nurse do prior to administering disulfiram to a client with a history of alcohol abuse? Assess the client's nutritional status. Assess when the client's last alcoholic beverage was consumed. Assess whether the client admits to a problem with alcohol. Assess the client's commitment to attend Alcoholics Anonymous (AA) meetings.

Assess when the client's last alcoholic beverage was consumed. The client must be alcohol free for 12 hours before starting therapy with disulfiram. Assessing the client's commitment to attend AA meetings, the client's perception of the problem, and nutritional status are all important interventions, but they aren't necessary prior to starting disulfiram.

A client has Prinzmetal's angina. Which type of medication should the nurse anticipate to reduce the risk of coronary artery spasms? Angiotensin-converting enzyme (ACE) inhibitor Calcium channel blocker Inotropic vasodilator Beta-adrenergic blocker

Calcium channel blocker A calcium channel blocker, such as diltiazem, is indicated for the management of Prinzmetal's angina. The class of medication would reduce the incidence of coronary artery spasm. A beta-adrenergic blocker, such as metoprolol, is used to treat angina by decreasing myocardial oxygen needs, and has no effect on coronary artery spasms. An ACE inhibitor, such as enalapril, is used to manage hypertension. An inotropic vasodilator, such as milrinone, is indicated for short-term IV therapy in heart failure.

A nurse is providing in-home management instructions to the parents of a child who is receiving desmopressin acetate (DDAVP). What is the most important instruction for the nurse to include? Cleanse skin with alcohol before application of the DDAVP dermal patch. Give DDAVP only when urine output begins to decrease. Increase the DDAVP dose if polyuria occurs just before the next scheduled dose. Call the healthcare provider if the child has an upper respiratory infection or allergic rhinitis.

Call the healthcare provider if the child has an upper respiratory infection or allergic rhinitis. Excessive nasal mucus, associated with upper respiratory infection or allergic rhinitis, may interfere with DDAVP absorption because it is given intranasally. Parents should be instructed to contact the health care provider for advice in altering the hormone dose during times when nasal mucus may be increased. The DDAVP dose should remain unchanged, even if the child is experiencing polyuria just before the next dose to avoid over medicating the child.

The nurse is providing care for a child diagnosed with hyperthyroidism. The child has been prescribed propylthiouracil to reduce thyroxine production. How can the nurse best address the potential adverse effects of this medication? Encourage small, frequent meals in order to prevent nausea. Closely monitor the child's white cell and platelet levels. Encourage the use of over-the-counter vitamin D supplements. Assess the child's sclerae for signs of jaundice.

Closely monitor the child's white cell and platelet levels. Thrombocytopenia and leukopenia are adverse effects of propylthiouracil therapy. There is no need for vitamin D supplements, and no added risk of jaundice or nausea.

Clients with schizophrenia often experience nonadherence to prescribed medication protocols. Nurses collaborate with these clients to develop a program of successful adherence. How are long-acting decanoate injections a helpful treatment option for these clients? Clients report significantly fewer side effects from decanoate injections than from oral versions of the same medicine. Decanoate injections improve adherence and sustained therapeutic drug levels despite possible client ambivalence. Clients generally do not recognize or report the side effects of injected medications. Clients who decline or miss their scheduled injection can receive a double dose the next time.

Decanoate injections improve adherence and sustained therapeutic drug levels despite possible client ambivalence. Long-acting decanoate injections are a good treatment option for clients with a known pattern of nonadherence, ambivalence about their treatment, and limited insight into their illness. These injections share the same side effects as oral forms of the drug. Injections are not implemented until oral medication tolerance and effectiveness is established. Clients do not necessarily recognize side effects or realize that they are related to medications or dosage. Decanoate injections have the same side effect profile as the oral version of the medication.

A client calls the clinic worried about experiencing new symptoms after taking antipsychotic medicine. The client reports persistent, uncontrollable restlessness of the limbs and head despite improvement in psychotic symptoms. What is the most appropriate intervention by the nurse? Advise the client to experiment with different dosages to see how that feels. Direct the client to see the provider for medication to address these side effects. Inform the client to ignore these symptoms because they will go away. Tell the client to go to the emergency room if blurred vision or fever develops.

Direct the client to see the provider for medication to address these side effects. Symptoms of tardive dyskinesia include tongue protrusion, lip smacking, chewing, blinking, grimacing, choreiform movements of limbs and trunk, and foot tapping. Primary prevention of tardive dyskinesia is achieved by using the lowest effective dose of a neuroleptic for the shortest time. However, with diseases of chronic psychosis such as schizophrenia, this strategy must be balanced with the fact that increased dosages are more beneficial in preventing recurrence of psychosis. If tardive dyskinesia is diagnosed, the causative drug should be discontinued. Blurred vision is a common adverse reaction of antipsychotic drugs and usually disappears after a few weeks of therapy. Restlessness is associated with akathisia. Sudden fever is a symptom of a malignant neurological disorder. The prescribing provider will make appropriate changes to meet the client's need. Clients should not ignore such symptoms, or adjust their own medication dosage.

Monoamine oxidase inhibitors (MAOIs) have been prescribed for a client with bulimia nervosa. What is the most important information for the nurse to give this client? Watch for bleeding and bruising. Do not eat foods that contain tyramines, such as cheese, cottage cheese, pickled herring, and salami. Drink several glasses of water with each dose. Do not drink water with meals. Call your provider if you have tremors or feel anxious or agitated.

Do not eat foods that contain tyramines, such as cheese, cottage cheese, pickled herring, and salami. The ingestion of foods containing tyramines can result in a hypertensive crisis. Water and other fluids may be taken with meals, but can limit the amount of food that can be eaten. Bleeding and bruising is not related to taking MAOIs. Symptoms such as tremors, anxiety, or agitation are also unrelated to taking MAOIs.

The nurse is planning to administer morphine 1 mg I.V. to a client who is reporting pain. The available vial is morphine 2 mg/1 ml. What is the most appropriate action by the nurse? Draw 1.0 ml of medication into a syringe, administer 0.5 ml of the medication, then ask another nurse to witness the waste of the remaining 0.5 ml. Draw 0.5 ml of medication into a syringe and place the vial with remaining medication into the sharps container. Draw 0.5 ml of medication into a syringe and ask another nurse to witness the waste of 0.5 ml of medication. Draw 0.5 ml of the medication into a syringe and place the remaining medication in the client's medication drawer for later use.

Draw 0.5 ml of medication into a syringe and ask another nurse to witness the waste of 0.5 ml of medication. Controlled substances such as morphine require close monitoring by federal law to control abuse by clients and healthcare workers. If a nurse is administering a partial dose of a controlled substance, the partial dose should be immediately discarded as per facility policy with a second nurse acting as a witness. Both nurses would document the disposal of the medication according to the facility policy and medication system in use. A controlled substance should never be disposed of in a sharps container as the container may be easily removed, allowing for diversion of the substance. Asking for a witness after the fact may raise suspicion of diversion. 2 mg/1 ml x 1 mg = 0.5 ml

A nurse is caring for a 7-year-old client receiving cyclophosphamide. In addition to administering mesna, which action should the nurse take? Limit the child's fluid intake. Transfuse platelets before administering the drug. Give the child cranberry juice to drink. Encourage the child to void frequently.

Encourage the child to void frequently. Hemorrhagic cystitis can result when the by-products of cyclophosphamide metabolism remain in the bladder; therefore, emptying the bladder at least every 2 hours when the child is awake can help prevent this painful condition. The child should be encouraged to void as soon as the urge is felt. Bacteria or low platelets do not cause the condition, so transfusing platelets and giving cranberry juice aren't correct. Fluids should not be limited. The child should be given liberal amounts of fluid, usually by I.V. infusion.

The serum calcium level remains low in a client with hypoparathyroidism despite taking calcium supplements. What should the nurse ask the client related to these findings? Are you also prescribed a thiazide diuretic along with the calcium supplements? Have you been taking your levothyroxine as prescribed in addition to calcium? Have you been taking vitamin D along with your calcium supplements? Are you eating a diet that consists of calcium-rich foods such as dairy products?

Have you been taking vitamin D along with your calcium supplements? A client with hypoparathyroidism has a decreased serum calcium level. Vitamin D enhances the absorption of calcium from the gastrointestinal tract and is the most important factor in improving the client's response to the calcium supplements. Even if the client increased dietary intake of calcium, without adequate vitamin D, this calcium would also be poorly absorbed. Thiazide diuretic therapy is linked to hypercalcemia, not hypocalcemia. Levothyroxine is given to treat hypothyroidism, not hypoparathyroidism.

A nurse is teaching a client with bipolar disorder about the drug carbamazepine. The nurse determines teaching was effective when the client states I need to have my blood counts checked periodically." "I can't take any other drugs while I am taking this one." "I will drink plenty of water so I don't develop kidney problems." "My hair will fall out after I take this drug for a few months."

I need to have my blood counts checked periodically." The most dangerous adverse effect of carbamazepine is bone marrow depression. Other medications may be taken with carbamazepine. Hair loss doesn't occur in clients taking carbamazepine. Clients who take lithium, not carbamazepine, must be closely monitored for nephrogenic diabetes insipidus. The interactions of all drugs must be monitored because some can either increase or decrease the blood level of carbamazepine.

A client who received massive packed red blood cell (PRBC) blood transfusions due to trauma has a potassium level of 7.1 mEq/L (7.1 mmol/L). Which medication should the nurse expect to administer? I.V. insulin oral spironolactone I.V. potassium chloride oral lisinopril

I.V. insulin The client is experiencing transfusion-associated hyperkalemia. Storing packed red blood cell increases the potassium concentration. I.V. regular insulin pushes potassium from the blood into the cell decreasing the serum potassium level. Severe cases require hemodialysis. I.V. potassium chloride and spironolactone, a potassium-sparing diuretic, will further increase the potassium. Angiotensin-converting enzyme (ACE) inhibitor such as lisinopril causes hyperkalemia.

The nurse is planning discharge teaching for a client who will continue taking the prescribed warfarin at home. What early symptoms of occult blood loss should the nurse teach the client? Decreased urine output and lightheadedness. Decrease in blood pressure. Increasing fatigue and shortness of breath. Decreased hemoglobin level.

Increasing fatigue and shortness of breath. Warfarin is an anticoagulant, so the priority teaching would include watching for signs of hemorrhage that can be occult (not visible). Early symptoms the nurse should focus on are ones the client will feel rather than signs that need to be measured by a laboratory such as hemoglobin levels. The earlier signs related to the reduction in oxygen carrying capacity include a feeling of fatigue and dyspnea. Later signs (not symptoms) include a drop in blood pressure (manifested by the symptom of lightheadedness) and decreased urine output which will only be obvious to the client once renal perfusion is significantly affected.

The nurse is preparing discharge instructions for a client taking lithium. What is the most important information for the nurse to give the client? Limit sodium intake to 2 to 3 grams per day to prevent fluid retention and increased blood pressure. Reduce fat and calorie intake to decrease the risk for weight gain associated with lithium. Maintain a consistent fluid intake each day, avoiding great fluctuations in volumes consumed. Include nonpharmacological treatments for depression such as vigorous cardiovascular exercise.

Maintain a consistent fluid intake each day, avoiding great fluctuations in volumes consumed. Clients taking lithium need to maintain a consistent fluid and sodium intake and not restrict either water or salt to avoid fluctuations that could alter lithium plasma levels. Vigorous exercise can increase water and sodium loss through perspiration and should not be done without the health care provider's guidance. Although lithium is associated with weight gain, not all clients are equally affected. Dieting can alter how lithium levels are balanced and should only be done with medical supervision.

What is the most important information for the nurse to include when teaching a client about metronidazole? Urine may develop a greenish tinge while the client is taking this drug. Mixing this drug with alcohol causes severe nausea and vomiting. Heart palpitations may occur and should be immediately reported. Breathlessness and cough are common adverse effects.

Mixing this drug with alcohol causes severe nausea and vomiting. When mixed with alcohol, metronidazole causes a disulfiram-like effect involving nausea, vomiting, and other unpleasant symptoms. Urine may turn reddish brown, not greenish, from the drug. Cardiovascular or respiratory effects are not associated with this drug.

A client with heart failure is given furosemide 40 mg I.V. daily. The morning serum potassium level is 2.8 mEq/L. Which nursing action is the most appropriate? Notify the health care provider and obtain additional orders. Question the health care provider about the dosage. Give the furosemide and get an order for sodium polystyrene sulfonate. Give 20 mg of the ordered dose and recheck the laboratory test results.

Notify the health care provider and obtain additional orders. Furosemide is a diuretic. Serum potassium is flushed from the body along with excess fluid. Notifying the health care provider of the low potassium level and getting an order for potassium chloride are appropriate actions before giving the furosemide. The nurse should not give half the dose without an order from the provider. Giving furosemide and sodium polystyrene sulfonate together would further lower the potassium level.

The nurse and occupational therapist are planning an outdoor volleyball game and picnic for eight mental health clients. What action should the nurse take for the two clients taking nortriptyline for depression? Provide protective clothing and apply sunscreen before going out. Omit the morning dose on the day of the picnic. Be aware that this drug can cause hypotension. Recognize that these clients may experience excessive thirst.

Provide protective clothing and apply sunscreen before going out. A common adverse effect of this drug is sensitivity to the sun. Protective clothing and sunscreen should be worn while the client is exposed to sunlight.

A 20-month-old toddler has been treated with permethrin for scabies. The toddler's parent asks, "Is this medication working? My child is still itching." Which response by the nurse is most appropriate? Pruritus is common in children under age 5 treated with permethrin. Pruritus can be present for weeks after treatment. Apply the drug every day until the rash and itching disappear. Stop treatment because the drug isn't safe for children under age 2.

Pruritus can be present for weeks after treatment. Pruritus may be present for weeks following treatment with permethrin. The drug is safe for use in infants as young as age 2 months. Treatment with permethrin can be safely repeated in 2 weeks. Pruritus is caused by secondary reactions of the mites.

A client with a subarachnoid hemorrhage is prescribed a 1,000 mg loading dose of I.V. phenytoin. What information is most important when administering this dose? Phenytoin should be administered through an I.V. catheter in the client's hand. Rapid phenytoin administration can cause cardiac arrhythmias. Therapeutic drug levels should be maintained between 20 and 30 mg/ml. Phenytoin should be mixed in dextrose in water before administration.

Rapid phenytoin administration can cause cardiac arrhythmias. Intravenous phenytoin should not exceed 50 mg/min, as rapid administration can depress the myocardium, causing lethal dysrhythmias. Therapeutic drug levels range from 10 to 20 mg/ml. Phenytoin is only compatible with normal saline, not dextrose in water. Phenytoin is very irritating to the blood vessels, and may cause purple glove syndrome when administered I.V. into a hand.

A nurse is teaching a client who received a dose of Rho(D) immune globulin at 28 weeks' gestation to prevent Rh isoimmunization. Which statement most accurately describes isoimmunization? Rh-positive fetal blood crosses into maternal blood, stimulating maternal antibodies. Rh-negative fetal blood crosses into maternal blood, stimulating maternal antibodies. Rh-positive maternal blood crosses into fetal blood, stimulating fetal antibodies. Rh-negative maternal blood crosses into fetal blood, stimulating fetal antibodies.

Rh-positive fetal blood crosses into maternal blood, stimulating maternal antibodies. Rh isoimmunization occurs when Rh-positive fetal blood cells cross into the maternal circulation and stimulate maternal antibody production. In subsequent pregnancies with an Rh-positive fetus, maternal antibodies may cross back into the fetal circulation and destroy the fetal blood cells.

The nurse is performing discharge teaching for a school-aged child who experienced an asthma attack. What is the most important information the nurse can provide this client about the prescription for budesonide? Use the medication before using a bronchodilator. This medication is used for acute asthma attacks. There is no need to use a spacer when taking this medication. Rinse the mouth after using this medication.

Rinse the mouth after using this medication. Oral candidiasis or thrush (a fungal infection of the throat) may occur in 1 in 25 persons who use budesonide without a spacer device on the inhaler. The risk is even higher with large doses, but is less in children than in adults. The child should be instructed to rinse the mouth after use and parents should be instructed to monitor the child's mouth for this. The medication should be given after using a bronchodilator to ensure maximum effectiveness. Corticosteroids should not be used for acute asthma attacks.

The health care provider is preparing a plan of care for a client with borderline personality disorder. Which medication would the nurse anticipate for this client? Antipsychotics, along with an antidepressant, will treat illusions, ideas of reference, paranoid thinking, anxiety, and hostility in clients. Monoamine oxidase inhibitors (MAOIs) work best because the effects are felt very quickly. Anxiolytics will reduce the anxiety and cognitive distortions that frequently occur in these clients. Selective serotonin reuptake inhibitors (SSRIs), along with an atypical antipsychotic, are used to treat mood instability and impulsivity.

Selective serotonin reuptake inhibitors (SSRIs), along with an atypical antipsychotic, are used to treat mood instability and impulsivity. Selective serotonin reuptake inhibitors and atypical antipsychotics are used to treat dysphoria, mood instability, and impulsivity in clients with borderline personality disorder. This is the best choice of medications for a client with borderline personality disorder. Monoamine oxidase inhibitors have food restrictions, and clients with borderline personality disorder would not comply with such restrictions. Antipsychotics are prescribed for psychotic behaviors such as illusions, ideas of reference, and paranoid thinking. Anxiolytics may be prescribed for clients with borderline personality disorder, but these medications are limited to addressing anxiety. Clients with borderline personality disorder experience symptoms other than anxiety.

A client has been receiving oxytocin to augment her labor. The nurse notes that contractions are lasting 100 seconds. Which immediate action should the nurse take? Turn the client on her left side. Stop the oxytocin infusion. Notify the provider. Monitor fetal heart tones as usual.

Stop the oxytocin infusion. Oxytocin should be withheld immediately, as it stimulates contractions. A contraction that continues for more than 90 seconds signals tetany and could lead to decreased placental perfusion and possibly uterine rupture. The nurse should monitor the fetal heart tones, stop the oxytocin, and notify the provider. The client should be turned on her left side to increase blood flow to the fetus, which can be decreased with tetany. This decreased blood flow can potentially compromise the fetus.

The nurse is instructing a client who will be discharged on anticoagulant therapy. What is the most important instruction for this nurse to include? Do not shave with an electric razor. Take ibuprofen or aspirin for pain. Take the anticoagulant at the same time each day. Eat green, leafy vegetables and salad daily.

Take the anticoagulant at the same time each day. It is important to take the anticoagulant at the same time each day to maintain an adequate blood level. An electric razor reduces the risk of cutting the skin. Avoid the use of standard razors. Avoid taking aspirin or ibuprofen because these drugs decrease clotting time. Eating a large amount of green, leafy vegetables that contain vitamin K will increase clotting time, thus requiring more anticoagulants.

A 5-year-old child diagnosed with cerebral palsy has just been prescribed oral baclofen. Which assessment finding by the nurse would indicate effective drug therapy? The child is exhibiting less spasticity. The child is better able to concentrate on mental activities. The child no longer sleeps during the daytime. The child has less frequent seizures.

The child is exhibiting less spasticity. Baclofen is a skeletal muscle relaxant that is effective in reducing overall spasticity. It is not an anti-seizure drug. Significant side effects of this drug are drowsiness and confusion, so this child would not be sleeping less, nor demonstrating a better ability to concentrate on mental activities.

A client experiencing alcohol withdrawal is prescribed lorazepam. The client's family asks the nurse about the purpose of the medication. What is the nurse's best response? The medication can help your family member relax and sleep. The lorazepam will reduce the your family member's symptoms of withdrawal. Lorazepam is a type of benzodiazepine medication. This medication will reduce your family member's cravings for alcohol.

The lorazepam will reduce the your family member's symptoms of withdrawal. Lorazepam is a short-acting benzodiazepine usually given for 1 week to ease the effects of alcohol withdrawal. It is not used to reduce cravings and, although it will help the client feel more relaxed and can enhance sleep, this is not the primary indication. Though it is a benzodiazepine, telling the family this information does not address the question of why the client has been prescribed this medication.

The nurse is caring for a frail, older adult client who is experiencing pain. At the client care meeting, the family asks if it is safe for the client to receive narcotics. The nurse is aware that the client is receiving hydromorphone hydrochloride for pain. What is the nurse's most appropriate response to this family? The narcotic is safe because it does not accumulate in the body. The drug does not cause any problems with breathing. This drug is similar to methamphetamine. The drug is not as strong as morphine.

The narcotic is safe because it does not accumulate in the body. Hydromorphone is a fast-acting narcotic analgesic drug and is a useful alternative to morphine or meperidine due to its short half-life. Morphine and meperidine can increase the risk of confusion in older adults. Hydromorphone is a synthetic drug similar to morphine with an 8 to 10 times more potent analgesic effect. Respiratory depression may occur, but is less frequent than with some other narcotics.

A client with suspected myasthenia gravis is to undergo a test with edrophonium. The client asks if edrophonium can be used to treat myasthenia gravis. What is the nurse's best response? With repeated edrophonium use, immunosuppression may occur. The short half-life of edrophonium makes it impractical for long-term use. Dry mouth and abdominal cramps may be intolerable adverse effects. It isn't available in an oral form.

The short half-life of edrophonium makes it impractical for long-term use. Edrophonium is not available in an oral form and the duration of action is 1 to 2 minutes, making it impractical for the long-term management of myasthenia gravis. Immunosuppression with repeated use is an adverse effect of steroid administration. Dry mouth and abdominal cramps are adverse effects of increased acetylcholine in the parasympathetic nervous system.

A nurse is evaluating the effectiveness of drug therapy for a client undergoing alcohol detoxification. Which finding would indicate that this client's drug therapy needs to be adjusted? The client has tolerated dosage increase during treatment. There are signs of toxicity from the drug. There are signs that the drug has prevented the occurrence of further problems. The medication has allowed the client to have appropriate interactions with staff.

There are signs of toxicity from the drug. If signs of toxicity exist during the detoxification period, drug therapy needs to be adjusted. Drug therapy is effective if it prevents further problems. Medication dosage may require adjustment to obtain the maximum benefit. If the drug enables the client to have therapeutic interactions with the staff, the client is benefiting from the therapy.

A client asks why a 2.5% hydrocortisone cream prescribed for eczema can't be used for longer than 1 week. What is the nurse's best response? If no improvement is seen after 1 week, an antibiotic will be prescribed. The drug loses its efficacy after prolonged use. If no improvement is seen, a stronger concentration will be prescribed. This reduces adverse effects, such as skin atrophy and fragility.

This reduces adverse effects, such as skin atrophy and fragility. Hydrocortisone cream should be used for brief periods to decrease adverse effects such as skin atrophy. The drug doesn't lose efficacy after prolonged use. A stronger concentration may not be prescribed if no improvement is seen, and an antibiotic would be inappropriate.

A client with bipolar disorder tells the nurse that she just found out she is pregnant, and is concerned because she takes lithium. What is the most important information for the nurse to provide to this client? Use of lithium usually results in serious congenital problems. Lithium causes severe urine retention and increased risk of toxicity. Thyroid problems can occur in the first trimester of the pregnancy. Women who take lithium are very likely to have a spontaneous abortion.

Use of lithium usually results in serious congenital problems. Use of lithium during pregnancy will result in congenital defects, especially cardiac defects. Thyroid problems don't occur in the first trimester of the pregnancy. In lithium toxicity, a condition called nontoxic goiter may occur. An adverse effect of lithium is polyuria, not urine retention. The rate of spontaneous abortion for women taking lithium is no greater than for nonusers.

A client has been prescribed neomycin and polymyxin B sulfates and hydrocortisone otic suspension, two drops in the right ear. What action is most important for the nurse take when instilling the medication? Verify the proper client and route. Position the client in the semi-Fowler's position. Warm the solution to prevent dizziness. Hold an emesis basin under the client's ear.

Verify the proper client and route. When giving medications, a nurse should follow the "rights" of medication administration, which include verification of right client and right route. The drops may be warmed to prevent pain or dizziness, but this action isn't essential. An emesis basin would be used for irrigation of the ear. The client should be placed in the lateral position for five minutes, not semi-Fowler's position, to prevent the drops from draining.

A client diagnosed with schizophrenia is receiving an antipsychotic medication. The provider has just prescribed benztropine. Which adverse reaction was this medication most likely prescribed for? hypertensive crisis tardive dyskinesia acute dystonia orthostatic hypotension

acute dystonia Benztropine is used as an adjunctive therapy in parkinsonism, and for all conditions and medications that produce extrapyramidal symptoms except tardive dyskinesia, which is permanent and untreatable. Its anticholinergic action reduces the extrapyramidal effects associated with antipsychotic drugs. Hypertensive crisis and orthostatic hypotension are not associated with extrapyramidal symptoms.

2/10/2017 at 1100 56-year-old client presents with sudden onset of stroke-like symptoms that started 45 minutes prior to admission. Presenting National Institute of Health Stroke Scale (NIHSS) Score = 20. Head CT negative for blood. Past medical history includes hypertension treated with an ACE inhibitor. Alteplase 68 mg IV, given over one hour.At the end of the alteplase infusion, the nurse notes that the client's tongue was swollen. What is the nurse's priority action? administer antihistamines, intravenous corticosteroids, and or epinephrine draw arterial blood gases and prepare for immediate intubation document this is a normal finding for the client have the client chew ice chips every 15 minutes

administer antihistamines, intravenous corticosteroids, and or epinephrine The client has orolingual angioedema, a rare allergic reaction to alteplase that is more common in those taking ACE inhibitors. The nurse should prepare to administer antihistamines, intravenous corticosteroids, or epinephrine per provider orders. When caught early, intubation can often be prevented. If the client is in respiratory distress, and swelling is significant, the nurse should prepare for immediate intubation to protect the airway. Arterial blood gases are not needed pre-intubation, as airway obstruction is the immediate concern. Ice chips are contraindicated. A swollen tongue is not a symptom of stroke.

After a thyroidectomy, the client develops a positive Trousseau's sign. What is the nurse's priority action? administer liothyronine therapy administer calcium gluconate administer potassium. chloride administer levothyroxine therapy

administer calcium gluconate Damage to the parathyroid glands can inadvertently occur during a thyroidectomy. This may cause a decrease in serum calcium, which causes muscle hyperexcitability and tetany. The treatment for a client who develops hypocalcemia and tetany following a thyroidectomy is calcium gluconate. Hypokalemia does not cause a positive Trousseau's sign. Decreased thyroid hormones will not cause tetany, however, the client will have to take thyroid replacement therapy following a thyroidectomy.

A client receives midazolam, 2 mg I.V., as sedation before bronchoscopy. Five minutes after the clients receives the drug, their respiratory rate drops to 4 breaths/min. What is the nurse's most appropriate action? administer naloxone administer phentolamine administer flumazenil administer protamine sulfate (Heparin antagonist)

administer flumazenil Flumazenil reverses the effects of benzodiazepines such as midazolam. Naloxone is used to reverse opioids, such as morphine. Protamine sulfate reverses the effects of heparin. Phentolamine is injected into the tissues to reverse the damaging effects of a dopamine infiltration.

The nurse is caring for a client postoperatively after having a low anterior resection of the colon 6 hours prior. The client rates incisional pain 6/10. The prescribed orders include morphine 1 to 2 mg IV every hour as needed for pain. The client is alert with vital signs within normal limits. How will the nurse best manage the client's pain? administer morphine 1 mg IV followed by morphine 2 mg IV in 1 hour administer morphine 2 mg IV and reassess pain level in 1 hour administer morphine 1 mg IV and repeat the dose in 1 hour administer morphine 1 mg IV and reassess pain level in 20 minutes

administer morphine 1 mg IV and reassess pain level in 20 minutes Morphine is an opioid analgesic. Prevention of respiratory depression and increased sedation begins with the administration of the lowest effective dose. To best manage the client's pain with dose range orders, the nurse would begin with the lowest prescribed dose and titrate as needed to achieve effective analgesia while minimizing side effects. After administering the lowest prescribed dose, the nurse would assess the client's pain level and response to the therapy in 20 minutes (morphine peaks in 20 minutes). Based on the client's response, the nurse would then administer additional morphine as necessary.

A client receiving phenothiazine has become restless and fidgety and is anxiously pacing the hallway. What adverse effect of phenothiazine would the nurse suspect, based on this behavior? tardive dyskinesia parkinsonian effects akathisia dystonia

akathisia Akathisia is an adverse effect of phenothiazines. Dystonia appears as excessive salivation, difficulty speaking, and involuntary movements of the face, neck, arms, and legs. Parkinsonian effects include a shuffling gait, hand tremors, drooling, rigidity, and loose arm movements. Tardive dyskinesia is characterized by odd facial and tongue movements.

The nurse is preparing to administer the initial dose of digoxin PO to a client. What is the nurse's priority assessment before administering this medication? respiratory rate radial heart rate apical heart rate blood pressure

apical heart rate Assessing the client's apical heart rate is essential before administering digoxin. The apex of the heart is the most accurate pulse point in the body. Blood pressure is only affected if the heart rate is too low, in which case the nurse would withhold digoxin. The radial heart rate can be affected by cardiac and vascular disease and; therefore, will not accurately depict the heart rate. Digoxin has no effect on respiratory function.

The nurse is caring for a client with Reye syndrome who is receiving pancuronium bromide. What is the most important intervention for the nurse to include in the plan of care? providing regular tactile stimulation placing the client in a supine position applying artificial tears as needed performing active range-of-motion (ROM) exercises

applying artificial tears as needed Pancuronium bromide suppresses the corneal reflex, making the eyes prone to irritation. Artificial tears prevent drying. Tactile stimulation isn't appropriate because it may elicit a pressure response. Active ROM exercises may cause an increase in pressure. The head of the bed should be elevated slightly, with the paralyzed client in a side-lying or semi-prone position to prevent aspiration and minimize intracranial pressure.

Which medication will the nurse administer to a client who experienced a thrombotic stroke two days ago? methylprednisolone aspirin alteplase acetaminophen

aspirin Aspirin interferes with platelet aggregation to prevent blood clots from forming or growing larger and is used in the treatment, and secondary prevention, of ischemic stroke due to thrombosis. Antiplatelet medication, such as aspirin, should be given by day two in the absence of a bleeding complication. Alteplase is a potent medication that breaks down blood clots. It is approved by the U.S. Food and Drug Administration (FDA) for treatment within three hours of the onset of ischemic stroke. When alteplase is given, the client should have a brain scan 24-hours post infusion, and prior to the initiation of anti-platelet therapy. Methylprednisolone is a steroid with mild anticoagulant properties and is not indicated in acute stroke.

A parent asks the nurse if medications can cause Reye syndrome. The nurse's most appropriate response is that Reye syndrome has been connected to acetaminophen. ibuprofen. aspirin. guaifenesin.

aspirin. Aspirin administration is associated with the development of Reye syndrome. Acetaminophen, ibuprofen, and guaifenesin have not been associated with the development of Reye syndrome. There has been a decreased incidence of Reye syndrome with the increased use of acetaminophen and ibuprofen for management of fevers in children.

The nurse is teaching the parents of a child with growth hormone deficiency how to administer growth hormone to their child. At what time should the nurse suggest administration of this medication? in the middle of the day after dinner first thing in the morning at bedtime

at bedtime Optimal therapeutic effect is typically achieved when the prescribed growth hormone is administered at bedtime. Pituitary release of growth hormone occurs during the first 45 to 90 minutes after the onset of sleep, so normal physiological release is mimicked with bedtime dosing.

Where is the best site for the nurse to assess a client's pulse prior to administering digoxin? the anterior aspect of the right arm at the antecubital fossa the left second intercostal space in the midclavicular line inner aspect of right wrist at the base of the thumb at the left fifth intercostal space, midclavicular line

at the left fifth intercostal space, midclavicular line The administration of digoxin requires the assessment of the client's apical pulse. The correct landmark for obtaining an apical pulse is the left fifth intercostal space at the midclavicular line. This is the point of maximum impulse, and the location of the left ventricular apex. The left second intercostal space in the midclavicular line is where pulmonic sounds are auscultated.

The laboratory has just notified the nurse that a client on the unit has a phenytoin level of 32 mg/dl. Which symptoms should the nurse anticipate from this client? ataxia and confusion sodium depletion tonic-clonic seizure urinary incontinence

ataxia and confusion A level of 32 mg/dl indicates phenytoin toxicity. Symptoms of toxicity include confusion and ataxia. Phenytoin doesn't cause hyponatremia, seizure, or urinary incontinence. Incontinence may occur during or after a seizure.

One hour after receiving pyridostigmine bromide for myasthenia gravis, a client reports difficulty swallowing and excessive respiratory secretions. What medication would the nurse anticipate to reverse the effects of pyridostigmine bromide? atropine additional pyridostigmine bromide acyclovir edrophonium

atropine These symptoms suggest cholinergic crisis or excessive acetylcholinesterase medication, typically appearing 45 to 60 minutes after the last dose of acetylcholinesterase inhibitor. Atropine, an anticholinergic drug, is used to antagonize acetylcholinesterase inhibitors. The other drugs are acetylcholinesterase inhibitors. Edrophonium is used for diagnosis, and pyridostigmine bromide is used to treat myasthenia gravis and would worsen these symptoms. Acyclovir is an antiviral and would not be used to treat these symptoms.

A client is receiving spironolactone to treat hypertension. Which instruction should the nurse provide? avoid salt substitutes take daily potassium supplements discontinue sodium restrictions eat foods high in potassium

avoid salt substitutes Because spironolactone is a potassium-sparing diuretic, the client should avoid salt substitutes because of their high potassium content. The client should also avoid potassium-rich foods and potassium supplements. To reduce fluid volume overload, sodium restrictions should continue.

A client, recovering from a spinal cord injury, has a great deal of spasticity. Which medication would the nurse anticipate to relieve spasticity? methylprednisolone hydralazine baclofen lidocaine

baclofen Baclofen is a skeletal muscle relaxant used to decrease spasms. It may be given orally or intrathecally. Hydralazine is an antihypertensive and afterload-reducing agent. Lidocaine is an antiarrhythmic and a local anesthetic agent. Methylprednisolone is an anti-inflammatory drug used to decrease spinal cord edema in the acute phase.

The nurse understands that certain medications protect the ischemic myocardium by blocking catecholamines and sympathetic nerve stimulation. Which class of medications serve this function? calcium channel blockers nitrates opioids beta-adrenergic blockers

beta-adrenergic blockers Beta-adrenergic blockers work by blocking beta receptors in the myocardium, reducing the response to catecholamines and sympathetic nerve stimulation. They protect the myocardium, and help reduce the risk of another infarction by decreasing the workload of the heart and decreasing myocardial oxygen demand. Calcium channel blockers reduce the workload of the heart by reducing contractility and vasodilatation; thus, lowering afterload. Opioids reduce myocardial oxygen demand, promote vasodilation, and decrease anxiety. Nitrates reduce myocardial oxygen consumption by decreasing left ventricular end-diastolic pressure and systemic vascular resistance.

The nurse is caring for a client prescribed a tocolytic agent. The nurse takes immediate action based on what assessment finding? bilateral crackles on lung auscultation maternal heart rate of 114 beats/min peripheral pulse strength of +2 blood glucose of 170 mg/dL (9.4 mmol/L)

bilateral crackles on lung auscultation Tocolytics are used to stop labor contractions. The most common adverse effect associated with the use of these drugs is pulmonary edema. Bilateral crackles on lung auscultation is a sign of pulmonary edema, and prompt action would be required. A serum glucose level of 170 mg/dL (9.4 mmol/L) is elevated and should be reported, but it is not life-threatening. Tocolytics may cause tachycardia and increased cardiac output with bounding arterial pulsations. A peripheral pulse strength of +2 indicates a slightly lower than normal level that is not an immediate cause for concern.

An 18-month-old is diagnosed with otitis media and the parent asks what can ease the client's pain. Which medication would the nurse anticipate for pain relief? children's liquid acetaminophen 5/ml q4h cetirizine 1.3 ml q4h amoxicillin trihydrate 20 mg/kg p0 q8h children's chewable acetylsalicylic acid one 80/mg q4h

children's liquid acetaminophen 5/ml q4h Children's acetaminophen is an anti-inflammatory and will decrease inflammation and pain. Children's acetylsalicylic acid is contradicted in all children due to the risk of Reye's Syndrome. Amoxicillin is an antibiotic used to treat bacterial infections of the middle ear and cetirizine is an antihistamine used to dry up secretions. Neither of these relieve pain.

Which medication would the nurse expect the provider to prescribe as prophylaxis against Pneumocystis carinii pneumonia for a client with leukemia? co-trimoxazole vincristine oral nystatin suspension prednisone

co-trimoxazole The most common cause of death from leukemia is overwhelming infection. P. carinii infection is lethal to a child with leukemia. As prophylaxis against P. carinii pneumonia, continuous low dosages of co-trimoxazole are typically prescribed. Oral nystatin suspension would be indicated for the treatment of thrush. Prednisone isn't an antibiotic, and increases susceptibility to infection. Vincristine is an antineoplastic agent.

The nurse notes an infant client with tetralogy of Fallot developed cyanosis and applies supplemental oxygen. What assessment finding should the nurse document as evidence of a positive response to oxygen therapy? increased blood pressure decreased agitation and restlessness improvement of finger clubbing decreased evidence of heart murmur

decreased agitation and restlessness Supplemental oxygen will help the infant breathe more easily and feel less anxious or agitated. Disappearance of the murmur and improvement of finger clubbing would not occur as a result of supplemental oxygen administration. Blood pressure is not increased by oxygen therapy. The nurse provides supportive treatment, such as reducing oxygen demand and increasing supply, until surgical correction can be performed.

The nurse is teaching the family of a client with a psychiatric disorder about traditional antipsychotic drugs and their effect on symptoms. Which symptom would be most responsive to these types of drugs? apathy social withdrawal delusions attention impairment

delusions Positive symptoms such as delusions, hallucinations, thought disorder, and disorganized speech respond to traditional antipsychotic drugs. The other options belong in a category of negative symptoms, including affective flattening, restricted thought and speech, apathy, anhedonia, asociality, and attention impairment. Negative symptoms are more responsive to the new atypical antipsychotics, such as clozapine, risperidone, and olanzapine.

A client has been prescribed digoxin to increase the heart's ability to contract effectively. The nurse is teaching a client about common side effects of digoxin. Of which side effects should this client be aware? Select all that apply. dizziness hyperactivity anxiety diarrhea headache

dizziness anxiety headache diarrhea Inotropic agents such as digoxin can trigger common side effects such as dizziness, anxiety, headache, and diarrhea. Changes in mood and alertness that include confusion and depression, rather than hyperactivity, may be observed. Weight loss is not associated with cardiac glycosides. The client should be instructed to notify the health care provider if any of these side effects become severe.

Which instruction should the nurse give to a client with prostatitis who is receiving double strength co-trimoxazole? if a sore mouth or throat develops, take the medication with milk or an antacid use a sunscreen of at least SPF-15 with para-aminobenzoic acid (PABA) don't expect improvement of symptoms for 7 to 10 days drink six to eight glasses of fluid daily while taking this medication

drink six to eight glasses of fluid daily while taking this medication Six to eight glasses of fluid daily are needed to prevent renal problems, such as crystalluria and stone formation. The symptoms should improve in a few days if the drug is effective. Sore throat and sore mouth are adverse effects that should be reported right away. The drug causes photosensitivity, but a PABA-free sunscreen should be used because PABA can interfere with the drug's action.

A newly admitted client diagnosed with delirium has a history of hypertension and anxiety. The client had been taking digoxin, furosemide, and diazepam. The nurse suspects that this client's impairment may be the result of: metabolic acidosis. drug intoxication. hepatic encephalopathy. opportunistic infection.

drug intoxication. Digoxin, furosemide, and diazepam have a propensity for producing delirium.

Which nursing intervention would help to decrease the adverse effects of radiation therapy on the gastrointestinal tract? avoiding mouthwashes to prevent irritation of mouth ulcers avoiding the use of antispasmodics encouraging fluids and a soft diet giving antiemetics when vomiting occurs

encouraging fluids and a soft diet Radiation therapy can cause adverse effects such as nausea and vomiting, anorexia, mucosal ulceration, and diarrhea. Antispasmodics are used to help reduce diarrhea. Encouraging fluids and a soft diet will help with anorexia. Antiemetics should be given before the onset of vomiting. Frequent mouthwashes are indicated to prevent mycosis.

A depressed client has been taking a selective serotonin reuptake inhibitor (SSRI) in the evening, and is upset because he cannot perform sexually due to erectile problems. What is the nurse's best response? take the drug with food or 8 oz of water engage in sexual activity prior to taking the drug monitor for low blood pressure on a daily basis stop taking the drug and notify the prescriber

engage in sexual activity prior to taking the drug A viable option is for the client to engage in sexual activity before taking his daily antidepressant medication. It is not appropriate to suggest stopping the medication. Monitoring the client's blood pressure and taking the drug with food or 8 oz of water will not address the erectile dysfunction experienced by the client.

Liquid oral iron supplements have been prescribed for a child. What is the most important information for the nurse to provide to this child's parents? give the medicine via a dropper or through a straw give the supplements with food decrease the dose if constipation occurs stop the medication if vomiting occurs

give the medicine via a dropper or through a straw Liquid iron preparations may temporarily stain the teeth. The drug should be given by dropper or through a straw. Iron supplements should be given between meals, when the presence of free hydrochloric acid is greatest. If vomiting occurs, supplementation should not be stopped, but it should be administered with food. Constipation can be decreased by increasing intake of fruits and vegetables.

The health care provider has prescribed salicylates for an older adult client with osteoarthritis to relieve pain. The nurse knows to assess the client for what potential adverse reaction? decreased calcium absorption hearing loss increased pain in joints increased bone demineralization

hearing loss Many older adults already have diminished hearing, and salicylate use can lead to further or total hearing loss. Salicylates do not increase pain in joints, decrease calcium absorption, or increase bone demineralization.

A definitive diagnosis of pulmonary embolism has been made for a client. Which medication would the nurse anticipate for this client? streptokinase heparin acyclovir warfarin

heparin Intravenous heparin is started once a pulmonary embolism is diagnosed to reduce clot formation. When a therapeutic level of heparin is established, warfarin is started. It can take up to three days before a therapeutic level of warfarin is achieved. Streptokinase is a fibrinolytic, usefulness in the management of pulmonary embolism. Acyclovir is an antiviral and is not prescribed after a pulmonary embolism.

Which assessment finding is expected in a client receiving bicalutamide and leuprolide for advanced prostate cancer? colicky pain abdominal distention acromegaly hot flashes

hot flashes Bicalutamide, a nonsteroidal antiandrogen, and leuprolide, a gonadotropin-releasing hormone agonist, decrease the production of testosterone. This helps decrease the production of cancer cells involved in prostate cancer. Because androgens are responsible for the development of male genitalia and secondary male sex characteristics, low androgen levels can cause genital atrophy, breast enlargement, and hot flashes. Abdominal distention, acromegaly, and colicky pain aren't caused by bicalutamide and leuprolide therapy.

A nurse is reviewing the healthcare provider's orders for a client admitted with systemic lupus erythematosus (SLE). Which medication would the nurse expect to find in this client's plan of care? dimenhydrinate ketoconazole hydroxychloroquine morphine

hydroxychloroquine Fatigue, photosensitivity and a "butterfly" rash on the face are all signs and symptoms of SLE. Hydroxychloroquine is used in the treatment of SLE to prevent inflammation. Pharmacological treatment of SLE also involves nonsteroidal anti-inflammatory drugs, corticosteroids, and immunosuppressive agents. Morphine is an opioid analgesic, ketoconazole is an antifungal agent, and dimenhydrinate is an antiemetic.

A client with chronic obstructive pulmonary disease (COPD) and cor pulmonale had been prescribed a loop diuretic to treat peripheral edema. The nurse should monitor the client closely for what side effect of loop diuretic therapy that could worsen the client's hypercapnia? hyperuricemia orthostatic hypotension hyponatremia hypokalemia

hypokalemia All the options offered are potential side effects of loop diuretics, but only hypokalemia would directly pose the risk for increasing hypercapnia. When potassium levels are low, hydrogen ions shift into the intracellular space to liberate potassium into the extracellular space, and this contributes to metabolic alkalosis. To compensate for metabolic alkalosis, hypoventilation occurs in an attempt to retain carbon dioxide (the respiratory acid) and decrease the client's pH. Therefore, hypokalemia can worsen hypercapnia. Diuretics must be used with caution in clients with COPD. However, diuretics may be prescribed to treat peripheral edema that results from right ventricular dysfunction and the resulting systemic venous congestion.

One hour after I.V. furosemide has been administered to a client with heart failure, a short burst of ventricular tachycardia appears on the cardiac monitor. Which electrolyte imbalance should the nurse suspect? hypokalemia hypernatremia hypocalcemia hypermagnesemia

hypokalemia Furosemide is a potassium-depleting diuretic that can cause hypokalemia. In turn, hypokalemia increases myocardial excitability, leading to ventricular tachycardia. Hypocalcemia, which slows conduction through the atrioventricular junction, can cause such bradyarrhythmias as atrioventricular block. Hypermagnesemia may lead to bradycardia, not tachycardia. Hypernatremia may cause sinus tachycardia as a result of water loss.

The nurse is preparing to administer I.V. insulin to a client diagnosed with diabetic ketoacidosis (DKA). What will the nurse monitor while the client is receiving this intervention? hypokalemia and hypoglycemia hypocalcemia and hyperkalemia hypernatremia and hypercalcemia hyperkalemia and hyperglycemia

hypokalemia and hypoglycemia The nurse should monitor for decreased potassium and decreased glucose. Hypoglycemia might occur if too much insulin is administered, or insulin is administered too quickly. Intravenous insulin forces potassium into cells, thereby lowering plasma levels of potassium. The client may have hyperkalemia prior to starting the insulin therapy, but hypokalemia will occur with insulin administration. Calcium and sodium levels should not be affected.

Which nifedipine-related side effect should the nurse be most concerned with when caring for a new stroke admission? hypocarbia hypotension dehydration weakness

hypotension Nifedipine is a calcium channel blocker used to lower blood pressure. It is avoided in acute stroke due to the potential of hypotension. Hypotension in acute ischemic stroke reduces brain perfusion and is associated with poor stroke outcomes. Treatment for an acute stroke includes permissive hypertension. Hypocarbia, dehydration, and weakness are not common side effects of nifedipine.

A client with bipolar disorder has been receiving lithium for two weeks. The client also takes chemotherapeutic drugs that cause them to feel nauseated and anorexic. It is most important for the nurse to assess this client for hypotonic reflexes with muscle weakness. oliguria and cardiac dysrhythmias. marked arthritis with joint tenderness. hyperpyrexia with double vision.

hypotonic reflexes with muscle weakness. Lithium alters sodium transport in nerve and muscle cells, slowing the speed of impulse transmission. Hypotonic reflexes and muscle weakness can be an adverse effect of lithium. Lithium has no known effect on body temperature, does not cause double vision, or arthritis with joint tenderness. Oliguria and symptoms of cardiac dysrhythmias occur late in severe lithium toxicity.

A client is prescribed a tricyclic antidepressant after other medications were ineffective. The nurse assesses for what outcome as evidence the new medication has been effective? reduction in purposeless movements improved cognitive functioning decreased daytime sleepiness moderate weight gain

improved cognitive functioning Tricyclic antidepressants (TCAs) are not recommended as first-line treatment for depression, because they carry more side effects than other classes such as selective serotonin reuptake inhibitors (SSRIs). If treatment with an SSRI is ineffective, the client may take a TCA to improve the symptoms of depression, such as increased energy level and interest in enjoyable activities, improved memory and cognitive functioning, and improved sleep. Weight gain is a side effect of TCAs but is not an indication of improvement of depressive symptoms. Drowsiness is also a side effect, so it is possible daytime sleepiness will increase, not decrease. Purposeless movements are not a symptom of depression.

A client with a large cerebral intracranial hemorrhage was given mannitol to decrease intracranial pressure (ICP). What therapeutic effect should the nurse anticipate from mannitol? evidence of rebound cerebral hypertension increased urine output normal blood urea nitrogen (BUN) and creatinine levels pupils that are bilaterally 7mm and nonreactive

increased urine output Mannitol promotes osmotic diuresis by increasing the pressure gradient in the renal tubules, thus increasing urine output. Fixed and dilated pupils are symptoms of increased ICP or cranial nerve damage, seen in herniation associated with a deteriorating cerebellar hemorrhage. No information is given about abnormal BUN and creatinine levels, or that mannitol is being given for renal dysfunction. Rebound cerebral hypertension is an adverse and undesired complication from ongoing mannitol use.

The nurse is providing discharge instructions for a client who is receiving chemotherapeutic medications. Which intervention is most important to prevent hemorrhagic cystitis? administering antibiotics increasing calcium intake increasing fluid intake administering antacids

increasing fluid intake Sterile hemorrhagic cystitis is an adverse effect of chemical irritation of the bladder from cyclophosphamide. It can be prevented by liberal fluid intake (at least one-and-a-half times the recommended daily fluid requirement). Antibiotics do not aid in the prevention of sterile hemorrhagic cystitis. Increasing calcium intake does not alter the risk of developing cystitis. Antacids would not be indicated for treatment.

A client is experiencing status asthmaticus. For which would the nurse anticipate an immediate order? I.V. beta-adrenergic agents inhaled Beta-2 adrenergic agonist inhaled corticosteroids oral corticosteroids

inhaled Beta-2 adrenergic agonist Inhaled beta-adrenergic agonists agents are the first line of therapy in status asthmaticus, as they help promote bronchodilation, which improves oxygenation. I.V. beta-adrenergic agents can be used, but must be carefully monitored because of their systemic effects. They are typically used when the inhaled beta-adrenergic agents do not work. Inhaled and oral corticosteroids are slow-acting, and their use won't reduce hypoxia in the acute phase.

A nurse is assessing a client with bipolar disorder. The client tells the nurse that the family health care provider prescribed lithium. Which symptom would indicate that the client is developing lithium toxicity? hypertension low urine output hyperexcitability lethargy

lethargy Nausea, vomiting, diarrhea, thirst, polyuria, lethargy, slurred speech, hypotension, muscle weakness, and fine hand tremors are signs of lithium toxicity.

Which antiparkinsonian drug can cause drug tolerance or toxicity if taken for too long? selegiline amantadine pergolide levodopa-carbidopa

levodopa-carbidopa Long-term therapy with levodopa-carbidopa can result in drug tolerance or toxicity manifested by confusion, hallucinations, or decreased drug effectiveness. The other drugs listed don't require the client to take a drug holiday

A laboring client in the latent stage of labor begins reporting pain in the epigastric area, blurred vision, and a headache. Which medication would the nurse anticipate for these symptoms? calcium gluconate oxytocin magnesium sulfate terbutaline

magnesium sulfate Magnesium sulfate is the drug of choice to treat hypertension of pregnancy because it reduces edema by causing a shift from the extracellular spaces into the intestines. It also depresses the central nervous system, which decreases the incidence of seizures. Terbutaline is a smooth muscle relaxant used to relax the uterus. Oxytocin is the synthetic form of the pituitary hormone used to stimulate uterine contractions. Calcium gluconate is the antagonist for magnesium toxicity.

A client being treated for an asthma attack is not responding and deemed to be in status asthmaticus. The nurse administers the prescribed treatments to accomplish what primary goal? a respiratory rate below 24 breaths per minute maintenance of independent ventilation an oxygen saturation above 95% breath sounds free of adventitia

maintenance of independent ventilation If the client is deemed to be experiencing status asthmaticus, this means there has been unsatisfactory response to typical treatment with bronchodilators and corticosteroids, necessitating additional treatments such as magnesium sulfate. Ultimately, the goal is to help the client maintain independent ventilation and avoid the need for invasive ventilatory support via intubation. The client may not be able to have clear breath sounds, a normal respiratory rate, or an oxygen saturation above 95% while in status asthmaticus. Furthermore, if the client were to develop severe hypercapnia, it is possible (though unlikely) to have a normal oxygen saturation and respiratory rate, and even clear breath sounds, but still require invasive ventilatory support.

A client with type 1 diabetes mellitus often skips their ordered dose of insulin. What priority information should the nurse give to this client regarding the omission of insulin doses? may lead to pancreatitis may cause diabetes insipidus may lead to ketoacidosis may cause hypoglycemic coma

may lead to ketoacidosis A client who fails to regularly take insulin is at risk for hyperglycemia, which could lead to diabetic ketoacidosis. Hypoglycemia would not occur because the lack of insulin would lead to increased levels of sugar in the blood. A client with chronic pancreatitis may develop diabetes, but insulin-dependent diabetes mellitus does not lead to pancreatitis. Diabetes insipidus isn't caused by alteration in insulin levels.

A client with joint pain, tenderness and swelling has been admitted to the hospital. A disease modifying anti-rheumatic drug (DMARD) is prescribed by the healthcare provider. Which medication should the nurse expect to administer? methotrexate aspirin prednisone ferrous sulfate

methotrexate Methotrexate is considered a first-line DMARD for most clients with rheumatoid arthritis (RA). NSAIDs, such as aspirin, cannot be tolerated. Ferrous sulfate is not used to treat RA. Prednisone may be used to control inflammation when NSAIDs cannot be used.

What is the nurse's priority action when administering phenytoin to a client intravenously? mix phenytoin with saline solution only withhold other anticonvulsants use only dextrose solution when flushing the I.V. catheter administer rapidly

mix phenytoin with saline solution only Phenytoin is only compatible with saline solutions. Dextrose will cause an insoluble precipitate to form. Phenytoin should be administered at a rate of less than 50 mg/min. There is no need to withhold additional anticonvulsants.

The nurse makes initial rounds for the clients. Five medications are scheduled for administration at the same time to five different clients. Which medication should the nurse administer first after initial rounds? ondansetron to a diabetic client reporting nausea naproxen to the client with rheumatoid arthritis a maintenance dose of digoxin to the client with congestive heart failure morphine sulfate to a client with a myocardial infarction reporting chest pain

morphine sulfate to a client with a myocardial infarction reporting chest pain Morphine sulfate relieves pain which immediately decreases myocardial oxygen demand and decreases preload and afterload pressure. The digoxin is a maintenance dose and does not elicit an immediate reaction. Though administration of naproxen and ondansetron are next in the order of urgency, they are not the priority.

A client reports pain one day after a colostomy. The nurse administers 4 milligrams morphine I.V., and reassesses the client 30 minutes later. The following is noted: Respiratory rate at 8 breaths/min. Nasal cannula on floor. Arterial blood gas (ABG) results are: pH, 7.23; PaO2, 58 mm Hg; PaCO2, 61 mm Hg; HCO3 24 mEq/l. Which factors most likely contribute to this client's ABG results? Morphine, the nasal cannula on the floor, and the colostomy colostomy, pain, and morphine pain, respiratory rate of 8 breaths/min, and the nasal cannula on the floor morphine, respiratory rate of 8 breaths/min, and the nasal cannula on the floor

morphine, respiratory rate of 8 breaths/min, and the nasal cannula on the floor This client has respiratory acidosis. Opioids can suppress respirations, causing retention of carbon dioxide. A PaO2 of 58 mm Hg indicates hypoxemia caused by the removal of the client's supplementary oxygen and decreased respiratory rate. Pain increases the rate of respirations, which causes a decrease in PaCO2. Colostomy drainage doesn't start until 2 to 3 days postoperatively, and this drainage would contribute to metabolic alkalosis.

Which assessment finding would advise the nurse of a need to change from the prescribed intranasal route to an injection of desmopressin acetate for a child with diabetes insipidus? severe coughing occasional nosebleeds pneumonia mucous membrane irritation

mucous membrane irritation Mucous membrane irritation, caused by a cold or allergy, can render the intranasal route unreliable. Severe coughing, pneumonia, and nosebleeds shouldn't interfere with the intranasal route.

What adverse reaction might the nurse observe after administering enteric-coated erythromycin to a client? increased appetite weight gain nausea and vomiting constipation

nausea and vomiting Erythromycin is an antibiotic. Common adverse effects include nausea, vomiting, diarrhea, abdominal pain, and anorexia. It should be given with a full glass of water and after meals, or with food, to lessen gastrointestinal symptoms.

A client with schizophrenia has been prescribed risperidone. The client's symptoms include hallucinations, delusions, and withdrawal. The nurse should explain that this medication will help improve: paranoid symptoms. negative symptoms only. positive symptoms only. negative and positive symptoms.

negative and positive symptoms. Risperidone targets both negative and positive symptoms. Positive symptoms include delusions, hallucinations, and bizarre behaviors. Negative symptoms indicate a loss or lack of normal functioning such as lack of motivation and social withdrawal.

A child with diabetic ketoacidosis is being treated for a blood glucose level of 738 mg/dl (41.0 mmol/L). The nurse should anticipate an order for: normal saline with regular insulin. normal saline with ultralente insulin. 5% dextrose in water with PZI insulin. 5% dextrose in water with NPH insulin.

normal saline with regular insulin. Short-acting regular insulin is the only insulin used for insulin infusions. Initially, normal saline is used until blood glucose levels are reduced, then a dextrose solution may be used to prevent hypoglycemia. Ultralente, NPH, and PZI insulins have a longer duration of action and shouldn't be used for continuous infusions.

A client diagnosed with schizophrenia has been taking haloperidol for 1 week when a nurse observes that the client's gaze is fixed on the ceiling. Which specific condition is the client exhibiting? akathisia oculogyric crisis tardive dyskinesia neuroleptic malignant syndrome

oculogyric crisis An oculogyric crisis involves the eyes fixated in one direction, typically in an upward gaze. Neuroleptic malignant syndrome causes increased body temperature, muscle rigidity, and altered consciousness. Akathisia is a restlessness that can cause pacing and tapping of the fingers or feet. Stereotyped involuntary movements, such as tongue protrusion, lip smacking, chewing, blinking, and grimacing characterize tardive dyskinesia.

A 2-year-old child has tested positive for tuberculosis (TB), and has been started on rifampin. The child's parents ask the nurse if there is any important information they should know about this medication. What important adverse effect should the nurse inform these parents about? orange body secretions decreased liver enzyme levels hyperactivity decreased bilirubin levels

orange body secretions Rifampin and its metabolites will turn urine, feces, sputum, tears, and sweat an orange color. This is not a serious adverse effect. Rifampin may also cause GI upset, headache, drowsiness, dizziness, visual disturbances, and fever. Liver enzyme and bilirubin levels increase because of hepatic metabolism of the drug. Parents should be taught the signs and symptoms of hepatitis and hyperbilirubinemia such as jaundice of the sclera or skin.

A client has been in early labor, with contractions every 10 to 12 minutes, for the past 12 hours with no progression. What medication should the nurse anticipate to help stimulate this client's uterine contractions? estrogen oxytocin terbutaline magnesium sulfate

oxytocin Oxytocin is the hormone responsible for stimulating uterine contractions. Pitocin, the synthetic form, has similar action, and may be given to some clients to induce or augment uterine contractions. Although estrogen plays a role in uterine contractions, it is not given in synthetic form to help uterine contractility. Progesterone has a relaxing effect on the uterus. Magnesium sulfate is used for maternal preeclampsia. It is not a tocolytic agent.

After undergoing a gastrectomy, a client develops pernicious anemia. Which route should the nurse use to administer cyanocobalamin (vitamin B12)? transdermal route buccal route parenteral route oral route

parenteral route A client who has undergone gastrectomy is no longer able to produce the intrinsic factor necessary for vitamin B12 absorption through the gastrointestinal tract; therefore, supplementation via parenteral route is required. This medication isn't available for buccal or transdermal routes.

Which medication would the nurse anticipate as the provider's treatment of choice for scarlet fever? amphotericin B prednisone penicillin acyclovir

penicillin The causative agent of scarlet fever is Group A beta-hemolytic streptococci, which is susceptible to penicillin. Erythromycin is used for penicillin-sensitive children. Anti-inflammatory drugs, such as prednisone, are not indicated for these clients. Acyclovir is used in the treatment of herpes infections. Amphotericin B is used to treat fungal infections.

The nurse anticipates that a client with intermittent claudication will receive which medication first? pentoxifylline warfarin analgesic heparin

pentoxifylline There are two main ways to treat claudication: medication and a surgical treatment called revascularization. Pentoxifylline is one of the medications that are used. It decreases blood viscosity, increases red blood cell flexibility, and improves flow through small vessels. By improving the client's circulation, oxygenated blood flow is increased. When arterial pain becomes severe, intermittent claudication progresses and causes pain. Analgesics would be prescribed to relieve pain. Warfarin and heparin are contraindicated, because they are anticoagulants.

The nurse is caring for a newborn with unrepaired transposition of the great vessels. Which medication should the nurse anticipate giving first for treatment of this defect? enalapril digoxin furosemide prostaglandin E1

prostaglandin E1 Prostaglandin E1 is necessary to maintain patency of the patent ductus arteriosus, and improve systemic arterial flow in children with inadequate intracardiac mixing. Digoxin, furosemide, and enalapril will treat heart failure when present.

A nurse is preparing to instill ear drops in a 28-year-old client with otitis externa. What is the correct procedure for instillation? pull the pinna up and back separate the palpebral fissures with a clean gauze pad pull the tragus up and back pull the pinna down and back

pull the pinna up and back To straighten the ear canal of an adult, the pinna is pulled up and back. Options 1 and 3 aren't appropriate methods for preparing the ear to receive eardrops. The palpebral fissures are in the eye.

A nurse is caring for a school-age client who is in the second percentile of height and weight for age as a result of an endocrine disorder. Which pharmacological intervention should the nurse anticipate? replacement with biosynthetic growth hormone treatment with testosterone or estrogen replacement with antidiuretic hormone (ADH) treatment with desmopressin acetate (DDAVP)

replacement with biosynthetic growth hormone The definitive treatment of growth hormone deficiency is the replacement of growth hormone (somatotropin) with biosynthetic somatotropin. This treatment is successful in 80% of affected children. Desmopressin acetate is used to treat diabetes insipidus. A deficiency of antidiuretic hormone causes diabetes insipidus, and isn't related to hypopituitarism. Testosterone or estrogen may be given during adolescence for normal sexual maturation, but neither is the definitive treatment for hypopituitarism.

A client who is receiving acetaminophen for osteoarthritis reports continuing pain. The healthcare provider prescribes celecoxib. What important information regarding this medication, should the nurse share with this client? if you miss a dose, take a double dose the next day report black and tarry stools to the health care provider don't take the medication with dairy products use a stool softener or fiber laxative daily to prevent constipation

report black and tarry stools to the health care provider Black and tarry stools are a sign of gastrointestinal (GI) bleeding, and may necessitate a medication change. Dairy products can help reduce GI irritation. The celecoxib dose should never be doubled. Constipation isn't an adverse effect of this medication.

A client with chronic alcohol use is admitted to the hospital for detoxification. Later that day, the client's blood pressure increases and the client is given lorazepam to prevent: seizure. anxiety reaction. fainting. stroke.

seizure. During detoxification from alcohol, changes in the client's physiological status, especially an increase in blood pressure, may indicate a possible seizure. Clients are treated with benzodiazepines to prevent this. Stroke, fainting, and anxiety aren't the primary concerns when withdrawing from alcohol.

A nurse is teaching a client diagnosed with a pulmonary embolism about the prescribed heparin therapy. The nurse determines that teaching has been effective when the client states heparin is given to slow the development of other clots. prevent clots from traveling to the lung. dissolve the clot. break up the pulmonary embolism.

slow the development of other clots. Heparin slows the development of other clots. It doesn't break up pulmonary embolisms or dissolve existing clots. Heparin doesn't stop clots from traveling to the lungs.

A client diagnosed with major depressive disorder three weeks ago reports feeling better since taking the prescribed antidepressant medication. The nurse prioritizes what assessment? substance abuse violent behavior suicidal ideation manic depression

suicidal ideation After a client has been on antidepressants and is feeling better, they commonly have the energy to harm themselves. Manic depression isn't treated with antidepressants. Nothing in the client's history suggests violent behavior. There are no signs or symptoms suggesting substance abuse.

An unemployed client without health insurance has not filled their prescription. Which assessment finding indicates that this client is not taking their levothyroxine as prescribed? temperature of 94° F (34.4° C) diarrhea rapid heart rate warm, dry, flushed skin

temperature of 94° F (34.4° C) Levothyroxine is prescribed for hypothyroidism, which causes a hypodynamic state. Failure to maintain levothyroxine therapy can lead to a low body temperature as well as slowing all metabolic processes. The other assessments indicate a hypermetabolic state, which could be symptomatic of an increase in thyroid hormones

The nurse receives an order to administer morphine to a client with an acute myocardial infarction. What is the purpose of this medication? to increase myocardial oxygen demand to decrease myocardial oxygen demand to decrease cardiac output to increase preload and afterload

to decrease myocardial oxygen demand Morphine will calm and relax the client and decrease respiratory rate, anxiety, and stress, thus decreasing myocardial oxygen demand. It doesn't have any effect on cardiac output or preload or afterload.

The family of a client in rehabilitation following heroin withdrawal asks a nurse why the client is receiving naltrexone. What is the nurse's best response? to help reverse withdrawal symptoms to decrease the client's memory of the withdrawal experience to take the place of detoxification with methadone to keep the client sedated during withdrawal

to help reverse withdrawal symptoms Naltrexone is an opioid antagonist and helps the client stay drug free. Keeping the client sedated during withdrawal isn't the reason for giving this drug. The drug doesn't decrease the client's memory of the withdrawal experience, and isn't used in place of detoxification with methadone.

The nurse is preparing to administer vasopressin to a client who has undergone a hypophysectomy. What is the purpose of the medication? to treat growth failure to prevent syndrome of inappropriate antidiuretic hormone (SIADH) to reduce cerebral edema and lower intracranial pressure to replace antidiuretic hormone (ADH) normally secreted from the pituitary

to replace antidiuretic hormone (ADH) normally secreted from the pituitary After hypophysectomy, or removal of the pituitary gland, the body can't synthesize ADH; therefore, vasopressin is administered. Somatropin or growth hormone is used to treat growth failure. SIADH results from excessive ADH secretion. Vasopressin is not used to treat cerebral edema.

A primipara who is Rho(D) negative has just given birth to a Rh-positive baby. The nurse is developing a plan of care. How should Rho(D) immune globulin be administered? to the neonate at the first well-baby visit to the client at her first postpartum visit in 6 weeks to the client within 3 days to the neonate within 3 days

to the client within 3 days Administering Rho(D) immune globulin to the client within 72 hours of birth prevents antibodies from forming that can destroy fetal blood cells in the next pregnancy. Rho(D) immune globulin isn't given to the baby. The client should not wait 6 weeks to receive Rho(D) immune globulin as antibodies will already have formed.

Before starting treatment for leukemia, a client receives I.V. fluids and allopurinol. These interventions reduce the risk for tumor lysis syndrome. pancytopenia. disseminated intravascular coagulation (DIC). mucositis.

tumor lysis syndrome. During chemotherapy for leukemia, tumor lysis syndrome may occur as cell destruction releases intracellular components, resulting in hyperuricemia. Large fluid quantities and allopurinol therapy help reduce the amount of uric acid that results from tumor lysis syndrome but don't stop the cell lysis. Although DIC, pancytopenia, and mucositis are possible chemotherapy complications, they're not treated with I.V. fluids and allopurinol.

A 47-year-old client has been taking prescribed medication for an intestinal ulcer. During a routine office visit for blood pressure monitoring, the client reports he is no longer able to have sexual intercourse with his spouse. The nurse determines that this is most likely the result of: stressful lifestyle. high blood pressure. ulcer medication. advancing age.

ulcer medication. Impotence in men is a lesser known side effect of ulcer medications prescribed for them. Impotence can occur at any time and is not age related. Elevated blood pressure itself doesn't cause impotence, but antihypertensive medication can produce this unwelcome side effect. It could contribute to the present difficulty in addition to possible side effects of the prescribed ulcer medication. Stress may cause erectile dysfunction, but there is no evidence that the client is under stress. Men are usually able to have an erection throughout their lives.

What is the most important assessment for the nurse to make when administering tamsulosin to a client with benign prostatic hyperplasia (BPH)? size of the prostate creatinine clearance serum testosterone level voiding pattern

voiding pattern The alpha-adrenergic blocker tamsulosin relaxes the smooth muscle of the bladder neck and prostate, so the urinary voiding symptoms of BPH are reduced in many clients. These drugs do not affect the size of the prostate, renal function, or the production or metabolism of testosterone.

Which physical assessment data would alert the nurse to a possible mild toxic reaction in a client receiving lithium? seizures increased appetite hypotension vomiting and diarrhea

vomiting and diarrhea Vomiting and diarrhea are signs of mild to moderate lithium toxicity. Hypotension and seizures occur with moderate to severe toxic reactions. Anorexia occurs with mild toxic reactions.

A client was admitted to the hospital because of a transient ischemic attack (TIA) secondary to atrial fibrillation. The nurse anticipates that the provider will prescribe digoxin. quinidine gluconate. warfarin. diltiazem.

warfarin. Atrial fibrillation may lead to the formation of mural thrombi, which may embolize to the brain. Warfarin will prevent further clot formation and prevent clot enlargement. The other drugs are used in the treatment and control of atrial fibrillation, but won't affect clot formation.


Set pelajaran terkait

AP Psychology - First semester Final

View Set

Chapter 12: Service Response Issues

View Set

Chapter 23: The Child with a Sensory or Neurological Condition.

View Set

Chapter 4 - General Issues in Research Design

View Set

Full List of Handel's Ice Cream Flavors

View Set

Ricci - Chapter 41: Nursing Care of the Child With a Cardiovascular Disorder prepu

View Set